4th Level Exam 3 Nclex style questions

Pataasin ang iyong marka sa homework at exams ngayon gamit ang Quizwiz!

A public health nurse is providing education on how to prepare for an outbreak of avian influenza. Which of the following should the nurse include in the teaching? A. Individuals should receive the seasonal flu vaccine to minimize the risk of infection B. Individuals should receive a one time influenza vaccine C. D.

(I forgot to copy the rest, but the answer had something to do with staying home and having 2-3 week supply of water.)

Which statement by the mother of a child with rheumatic fever (RF) shows she has an understanding of prevention for her other children? SATA 1. "Whenever one of them gets a sore throat, I will give that child an antibiotic." 2. "There is no treatment. It must run its course." 3. "If their culture is positive for group A streptococcus, i will give them their antibiotic." 4. "If their culture is positive for staphylococcus A, I will give them their antibiotic.

1&3 1. "Whenever one of them gets a sore throat, I will give that child an antibiotic." 3. "If their culture is positive for group A streptococcus, i will give them their antibiotic." ***No antibiotics given if viral, so the sore throat has to be caused by bacterial (strep) to get antibiotics.

Hypoxic spells in the infant with a congenital heart defect (CHD) can cause which of the following? Select all that apply. 1. Polycythemia. 2. Blood clots. 3. Cerebrovascular accident. 4. Developmental delays. 5. Viral pericarditis. 6. Brain damage. 7. Alkalosis

1, 2, 3, 4, 6. - (1-3)Hypoxia causes polycythemia, which can lead to increased blood viscosity, which can lead to blood clots and a stroke. (4) Developmental delays can be caused by multiple hospitalizations and surgeries, the child usually catches up to the appropriate level. (6) Hypoxia-blood clots- stroke = brain damage. **Hypoxic episodes cause acidosis, not alkalosis

Which are the most serious complications for a child with Kawasaki disease (KD)? Select all that apply. 1. Coronary thrombosis. 2. Coronary stenosis. 3. Coronary artery aneurysm. 4. Hypocoagulability. 5. Decreased sedimentation rate. 6. Hypoplastic left heart syndrome.

1, 2, 3. Thrombosis, stenosis, and aneurysm affect blood vessels. The child with KD has hypercoagulability and an increased sedimentation rate due to inflammation.

Tetralogy of Fallot (TOF) involves which defects? Select all that apply. 1. Ventricular septal defect (VSD). 2. Right ventricular hypertrophy. 3. Left ventricular hypertrophy. 4. Pulmonic stenosis (PS). 5. Pulmonic atresia. 6. Overriding aorta. 7. Patent ductus arteriosus (PDA).

1, 2, 4, 6. - TOF is a congenital defect with 4 defects - a ventricular septal defect, right ventricular hypertrophy, pulmonary valve stenosis, and overriding aorta.

What can an electrocardiogram (ECG) detect? Select all that apply. 1. Ischemia. 2. Injury. 3. Cardiac output (CO). 4. Dysrhythmias. 5. Systemic vascular resistance (SVR). 6. Occlusion pressure. 7. Conduction delay.

1, 2, 4, 7. 1. An electrocardiogram can indicate ischemia of the heart muscle. 2. An electrocardiogram can indicate injury to the heart muscle. 4. An electrocardiogram can show dysrhythmias. 7. An electrocardiogram does show conduction delays. (3,5,6 - echo)

The following are examples of acquired heart disease. Select all that apply. 1. Infective endocarditis. 2. Hypoplastic left heart syndrome. 3. Rheumatic fever (RF). 4. Cardiomyopathy. 5. Kawasaki disease (KD). 6. Transposition of the great vessels.

1, 3, 4, 5. (endocarditis, RF, cardiomyopathy, KD) 1. Infective endocarditis is an example of an acquired heart problem. 3. RF is an acquired heart problem. 4. Cardiomyopathy is an acquired heart problem. 5. KD is an acquired heart problem

Exposure to which illness should be a cause to discontinue therapy and substitute dipyri- damole (Persantine) in a child receiving aspirin therapy for Kawasaki disease (KD)? 1. Chickenpox or influenza. 2. E. coli or staphylococcus. 3. Mumps or streptococcus A. 4. Streptococcus A or staphylococcus

1. Both chickenpox and influenza are viral in nature, so consider stopping the aspirin because of Reyes syndrome

A child born with Down syndrome should be evaluated for which associated cardiac manifestation? 1. Congenital heart defect (CHD). 2. Systemic hypertension. 3. Hyperlipidemia. 4. Cardiomyopathy.

1. CHD is found often in children with Down syndrome, **most notably the Atrioventricular Canal Defect (AVC)

A nursing action that promotes ideal nutrition in an infant with congestive heart failure (CHF) is: 1. Feeding formula that is supplemented with additional calories. 2. Allowing the infant to nurse at each breast for 20 minutes. 3. Providing large feedings every 5 hours. 4. Using firm nipples with small openings to slow feedings.

1. Formula can be supplemented with extra calories, either from a commercial supplement, such as Polycose, or from corn syrup. Calories in formula could increase from 20 kcal/oz to 30 kcal/oz or more.

Which patient could require feeding by gavage? 1. Infant with congestive heart failure (CHF). 2. Toddler with repair of transposition of the great vessels. 3. Toddler with Kawasaki disease (KD) in the acute phase. 4. School-age child with rheumatic fever (RF) and chorea

1. The child may experience increased cardiac demand while feeding. Feedings by gavage eliminate that work and still provide high-calorie intake for growth.

Do not administer Digoxin (lanoxin) if heart rate is less than ____ in an infant (numeric response only)

100 (BUT... for the test, The dr will write an order to hold at his/her specified heart rate)

44) Treatment for congestive heart failure (CHF) in an infant began 3 days ago and has included digoxin and furosemide. The child no longer has retractions, lungs are clear, and HR is 60 bpm while the child sleeps. The nurse is confident that the child has disuresed successfully and had good renal perfusion when the nurse notes the child's urine output is: 1. 0.5 cc/kg/hr. 2. 1 cc/kg/hr. 3. 30 cc/hr. 4. 1 oz/hr.

2 - 1cc/kg/hr

Which finding might delay a cardiac catheterization procedure on a 1-year-old? 1. 30th percentile for weight. 2. Severe diaper rash. 3. Allergy to soy. 4. Oxygen saturation of 91% on room air.

2. A child with severe diaper rash has potential for infection if the interventionist makes the standard groin approach.

A child has been seen by the school nurse for dizziness since the start of the school term. It happens when standing in line for recess and homeroom. The child now reports that she would rather sit and watch her friends play hopscotch because she cannot count out loud and jump at the same ±me. When the nurse asks her if her chest ever hurts, she says yes. Based on this history, the nurse suspects that she has: 1. Ventricular septal defect (VSD). 2. Aortic stenosis (AS). 3. Mitral valve prolapse. 4. Tricuspid atresia

2. AS can progress, and the child can develop exercise intolerance that can be better when resting

A 10-year-old has undergone a cardiac catheterization. At the end of the procedure, the nurse should first assess: 1. Pain. 2. Pulses. 3. Hemoglobin and hematocrit levels. 4. Catheterization report.

2. Checking for pulses, especially in the canulated extremity, would assure perfusion to that extremity and is the priority post procedure.

The mother of a toddler reports that the child's father has just had a myocardial infarction (MI). Because of this information, the nurse recommends the child have a(n): 1. Electrocardiogram. 2. Lipid profile. 3. Echocardiogram. 4. Cardiac catheterization.

2. Current recommendations are for a lipid profile in children over 2 years with a first- or second-degree relative with stroke, myocardial infarction, angina, or sudden cardiac death. Also screen if parent, sibling, or grandparent has cholesterol of 240 mg/dL or greater

Congenital heart defects (CHDs) are classified by which of the following? Select all that apply. 1. Cyanotic defect. 2. Acyanotic defect. 3. Defects with increased pulmonary blood flow. 4. Defects with decreased pulmonary blood flow. 5. Mixed defects. 6. Obstructive defects. 7. Pansystolic murmur

3, 4, 5, 6. . Heart defects are now classified as defects with increased or decreased pulmonary blood flow, mixed or obstruction.

Which drug should not be used to control secondary hypertension in a sexually active adolescent female who uses intermittent birth control? 1. Beta blockers. 2. Calcium channel blockers. 3. ACE inhibitors. 4. Diuretics.

3. ACE inhibitors and angiotensin II receptor blockers can cause birth defects. The others are not teratogenic

Which statement by the mother of a child with rheumatic fever (RF) shows she has good understanding of the care of her child? 1. "I will apply heat to his swollen joints to promote circulation." 2. "I will have him do gentle stretching exercises to prevent contractures." 3. "I will give him the aspirin that is ordered for pain and inflammation 4. I will apply cold packs to his swollen joints to reduce pain

3. Aspirin is the drug of choice for treatment of RF

Which medication should the nurse give to a child diagnosed with transposition of the great vessels? 1. Ibuprofen. 2. Betamethasone. 3. Prostaglandin E. 4. Indocin

3. Prostaglandin E inhibits closing of the PDA, which connects the aorta and pulmonary artery.

Which assessment indicates that the parent of a 7-year-old is following the prescribed treatment for congestive heart failure (CHF)? 1. HR of 56 bpm. 2. Elevated RBC count. 3. 50th percentile height and weight for age. 4. Urine output of 0.5 cc/kg/hr.

3. The 50th percentile height and weight for age shows good growth and development, indicating good nutrition and perfusion.

On examination, a nurse hears a murmur at the left sternal border (LSB) in a child with diarrhea and fever. The parent asks why the pediatrician never said anything about the murmur. The nurse explains: 1. "The pediatrician is not a cardiologist." 2. "Murmurs are difficult to detect, especially in children 3. The increased CO of the fever increases the intensity of the murmur, making it easier to hear. 4. We need to refer the child to an interventional cardiologist

3. The increased CO of the fever increases the intensity of the murmur, making it easier to hear.

Which plan would be appropriate in helping to control congestive heart failure (CHF) in an infant? 1. Promoting fluid restriction. 2. Feeding a low-salt formula. 3. Feeding in semi-Fowler position. 4. Encouraging breast milk.

3. The infant has a great deal of difficulty feeding with CHF, so even getting the maintenance fluids is a challenge. The infant is fed in the more upright position so fluid in the lungs can go to the base of the lungs, allowing better expansion. **carseat position

In which congenital heart defect (CHD) would the nurse need to take upper and lower extremity BPs? 1. Transposition of the great vessels. 2. Aortic stenosis (AS). 3. Coarctation of the aorta (COA). 4. Tetralogy of Fallot (TOF).

3. With COA there is narrowing of the aorta, which increases pressure proximal to the defect (upper extremities) and decreases pressure distal to the defect (lower extremities). There will be high BP and strong pulses in the upper extremities and lowerthan- expected BP and weak pulses in the lower extremities.

After having several Stokes-Adams attacks over 4 months, a client reluctantly agrees to implantation of a permanent pacemaker. Before discharge, the nurse reviews pacemaker care and safety guidelines with the client and spouse. Which safety precaution is appropriate for a client with a pacemaker? 1. Stay at least 2 away from microwave ovens 2. Never engage in activities that require vigorous arm and shoulder movement 3. Avoid going through airport metal detectors 4. Avoid having magnetic resonance imaging (MRI).

4. A client with a pacemaker should avoid having an MRI because the magnet may disrupt pacemaker function and cause injury to the client.

Which statement by a parent of an infant with congestive heart failure (CHF) who is being sent home on digoxin indicates the need for further education? 1. "I will give the medication at regular 12-hour intervals." 2. "If he vomits, I will not give a make-up dose." 3. "If I miss a dose, I will not give an extra dose 4. I can mix the digoxin with his formula to make it taste better"

4. If the medication is mixed in his formula, and he refuses to drink the entire amount, the digoxin dose will be inadequate.

A child with laryngotracheobronchitis (croup) is placed in a cool mist tent. The mother becomes concerned because the child is frightened, consistently crying and trying to climb out of the tent. Which is the most appropriate nursing action? 1. Tell the mother that the child must stay in the tent. 2. Place a toy in the tent to make the child feel more comfortable. 3. Call the health care provider and obtain a prescription for a mild sedative. 4. Let the mother hold the child and direct the cool mist over the child's face.

4. Let the mother hold the child and direct the cool mist over the child's face.

Which of the following conditions most commonly results in CAD? A. Atherosclerosis B. DM C. MI D Renal failure

A Atherosclerosis, or plaque formation, is the leading cause of CAD. DM is a risk factor for CAD but isn't the most common cause. Renal failure doesn't cause CAD, but the two conditions are related. Myocardial infarction is commonly a result of CAD.

A nurse is assessing the neurovascular of a client who has returned to the surgical nursing unit 4 hours ago after undergoing aortoiliac bypass graft. The affected leg is warm, and the nurse notes redness and edema. The pedal pulse is palpable and unchanged from admission. The nurse interprets that the neurovascular status is: A Normal because of increased blood flow through the leg B Slightly deteriorating and should be monitored for another hour C Moderately impaired, and the surgeon should be called D Adequate from an arterial approach, but venous complications are arising

A An expected outcome of surgery is warmth, redness, and edema in the surgical extremity because of increased blood flow. (**TEST 4)

A nurse is providing information to a group of clients at a local community center about TB. Which of the following clinical manifestations should be included in the teaching? SATA a. persistent cough b. weight gain c. fatigue d. night sweats e. purulent sputum

A, C, D, E. Weight loss not gain.

Which signs cause the nurse to suspect cardiac tamponade after a client has cardiac surgery? Check all that apply. A Tachycardia B Hypertension C Increased CVP D Increased urine output E Jugular vein distention

A, C, E Blood in the pericardial sac compresses the heart so the ventricles cannot fill; this leads to a rapid thready pulse. Tamponade causes hypotension and a narrowed pulse pressure. As the tamponade increases, pressure on the heart interferes with the ejection of blood from the left ventricle, resulting in an increased pressure in the right side of the heart and the systemic circulation. As the heart because more inefficient, there is a decrease in kidney perfusion and therefore urine output. The increased venous pressure caused JVD.

Which of the following instructions should be included in the discharge teaching for a patient discharged with a transdermal nitroglycerin patch? A "Apply the patch to a non hairy, nonfatty area of the upper torso or arms." B "Apply the patch to the same site each day to maintain consistent drug absorption." C "If you get a headache, remove the patch for 4 hours and then reapply." D "If you get chest pain, apply a second patch right next to the first patch."

A. A nitroglycerin patch should be applied to a non hairy, nonfatty area for the best and most consistent absorption rates. Sites should be rotated to prevent skin irritation, and the drug should be continued if headache occurs because tolerance will develop. Sublingual nitroglycerin should be used to treat chest pain.

After open-heart surgery a client develops a temperature of 102*F. The nurse notifies the physician, because elevated temperatures: A Increase the cardiac output B May indicate cerebral edema C May be a forerunner of hemorrhage D Are related to diaphoresis and possible chilling

A. Temperatures of 102*F or greater lead to an increased metabolism and cardiac workload

Which individuals would the nurse identify as having the highest risk for CAD? A. A 45-year-old depressed male with a high-stress job B. A 60-year-old male with below normal homocysteine levels C. A 54-year-old female vegetarian with elevated high-density lipoprotein (HDL) levels D. A 62-year-old female who has a sedentary lifestyle and body mass index (BMI) of 23 kg/m2

A. A 45-year-old depressed male with a high-stress job The 45-year-old depressed male with a high-stress job is at the highest risk for CAD. Studies demonstrate that depression and stressful states can contribute to the development of CAD. Elevated HDL levels and low homocysteine levels actually help to prevent CAD. Although a sedentary lifestyle is a risk factor, a BMI of 23 kg/m2 depicts normal weight, and thus the patient with two risk factors is at greatest risk for developing CAD.

The nurse explains to the parents of a 5 year old with a VSD that a cardiac cath has been scheduled to: A. Identify the specific location of the defect B. Determine the degree of cardiomegaly present C. Confirm the presence of a pansystolic murmur D. Establish the presence of ventricular hypertrophy

A. A cardiac catheterization will identify the exact location of the VSD as well as assess pulmonary pressures.

The Causative agent of Tuberculosis is said to be: A Mycobacterium Tuberculosis B Hansen's Bacilli C Bacillus Anthracis D Group A Beta Hemolytic Streptococcus

A. Mycobacterium Tuberculosis

When planning emergent care for a patient with a suspected MI, what should the nurse anticipate administrating? A. Oxygen, nitroglycerin, aspirin, and morphine B. Oxygen, furosemide (Lasix), nitroglycerin, and meperidine C. Aspirin, nitroprusside (Nipride), dopamine (Intropin), and oxygen D. Nitroglycerin, lorazepam (Ativan), oxygen, and warfarin (Coumadin)

A. Oxygen, nitroglycerin, aspirin, and morphine In order - Aspirin Oxygen, Nitro, Morphine The American Heart Association's guidelines for emergency care of the patient with chest pain include the administration of oxygen, nitroglycerin, aspirin, and morphine. These interventions serve to relieve chest pain, improve oxygenation, decrease myocardial workload, and prevent further platelet aggregation. The other medications may be used later in the patient's treatment.

The nurse is counseling a woman who is beginning anti tubercular therapy with rifampin. The patient also takes an oral contraceptive. Which statement by the nurse is most accurate regarding potential drug interactions? a. You will need to switch to another form of birth control while you are taking rifampin. b. your birth control pills will remain effective while you are taking rifampin. c. you will need to take a stronger dose of birth control pills while you are on the rifampin. d. you will need to abstain from sexual intercourse while on the rifampin to avoid pregnancy.

A. Rifampin "RED" and not "REDy" to get pregnant. Counteracts birth control.

A nurse is assessing a 5 year old with a history of heart failure. Which finding indicates that the child has adequate cardiac output? A. Urine output of 30 mL/h B. Heart rate of 120 beats/min C. Cap refill time of 10 to 15 sec D. Bilateral crackles heard on auscultation.

A. The minimal hourly urine output should be at least 30 mL/hr for an adult or a child. The normal heart rate for a 5 yr old is 70 to 90 bts minute. Adequate cap refill time is 3 to 5 seconds. Crackles are an abnormal finding and may indicate hypervolemia, or excess circulating fluid volume, and heart failure.

When the patient with known angina pectoris complains that he is experiencing chest pain more frequently even at rest, the period of pain is longer, and it takes less stress for the pain to occur, the nurse recognizes that the patient is describing A. Unstable angina. B. Refractory angina. C. Variant angina. D. Intractable angina

A. Unstable angina is also called crescendo or pre-infarction angina and indicates the need for a change in treatment.

In the fetal heart, the Foramen Ovale allows blood to enter the Left _____ from the Right Atrium.

ATRIUM.. The foramen ovale is a normal cardiac structure found in all newborns and can be best described as a "door" between the right and left atria.

A client is brought into the ER with a third degree heart block after experiencing an acute anterior myocardial infarction. Which of the following interventions is the priority on an emergency basis? A) Temporary Pacemaker B) Administer Lidocaine C) Cardioversion D) Administer Atropine

Ans. A) A third degree heart block is a lethal rhythm. It is the complete blockage of the atrial impulses into the ventricles. The block may be at the AV node, Bundle of His resulting in the atrai and ventricles beating independently of each other. The atrial rate is usually normal while the ventricular is slow and below 55 bpm. The causes may be AMI, CAD, surgery, aging, or drug toxicity such as digoxin, procainamide, or verapam

If the pacemaker spike is seen right before the P wave, what does this indicate?

Atrial pacing

Which of the following signs and symptoms indicate pacemaker failure? a) excessive thirst b) prolonged hiccups c) flushing of the skin d) increased urine output

B - prolonged hiccups indicate pacemaker failure. Other signs and symptoms of pacemaker failure are dysrhythmias, dizziness, faintness, chest pain, shortness of breath, increase or decrease in apical rate.

A 1 year old child is diagnosed with a congenital heart defect after cardiac catheterization. His parents express concern about activities at home. Which is the nurse's best response? A. "You'll have to establish strict discipline so that he learns what he can't do". B. "Allow him to play and be active as long as he doesn't get fatigued". C. "He'll only be able to play by himself." D. "Discipline and limit-setting need to be relaxed to reduce his stress and crying."

B Parents should encourage normalcy within the limits of the child's condition. The child needs to have appropriate limits and discipline, but being too strict or overindulging the child makes it hard for him to learn acceptable behavior. A 1 year old child is beginning to explore his world and needs to have activities with other children.

A nurse is working with a group of parents whose infants have died from sudden infant death syndrome (SIDS). Which statement by a parent indicates the need for further teaching? A. "I feel so guilty because my baby died" B. "If I'd checked on my baby in the middle of the night he wouldn't have died" C. "Parents should take all pillows and stuffed animals out of a baby's crib" D. "it would probably be a good idea if I learned CPR"

B.

A patient is being discharged after the insertion of a permanent pacemaker. Which statement made by the patient indicates an understanding regarding appropriate self-care? A. "Every morning I will perform arm and shoulder stretches." B. "Each day I'll take my pulse and record it in a log." C. "I'll have to get rid of my microwave oven." D. "I won't be able to use my electric blanket anymore."

B.

Which of the following actions is the first priority care for a client exhibiting signs and symptoms of coronary artery disease? A Decrease anxiety B Enhance myocardial oxygenation C Administer sublignual nitroglycerin D Educate the client about his symptoms

B. Enhancing mocardial oxygenation is always the first priority when a client exhibits signs and symptoms of cardiac compromise. Without adequate oxygen, the myocardium suffers damage. Sublingual nitorglycerin is administered to treat acute angina, but its administration isn't the first priority. Although educating the client and decreasing anxiety are important in care delivery, nether are priorities when a client is compromised.

A nurse is teaching the parents of a child with tetralogy of Fallot about hypercyanotic spells ("tet spells"). When a spell occurs, the parents should: A. Call the physician immediately call the physician immediately B. Use a calm, comforting approach C. Lay the child in the supine position D. Take the child to the nearest emergency dept.

B. Hypercyanotic spells ("tet spells"), in which a child has an extreme bluish discoloration of the skin and mucous membranes, are commonly seen in children with tetralogy of Fallot (a condition with four cardiac anomalies: VSD, pulmonic stenosis, an overriding aorta, and right ventricular hypertrophy). The parents should maintain a calm, comforting approach and place the child in the knee-chest position. It isn't necessary to call the physician, and the spells aren't considered a medical emergency unless profound hypoxia occurs.

A 16 year old is admitted to the emergency department with complaints of sudden, severe chest pain. He says that he didn't experience any recent trauma to the chest. What should the nurse next ask about? A. Exercise and weight lifting B. Cocaine use C. Smoking D. Family history of myocardial infarction (MI)

B. The nurse should next ask about cocaine use. Cocaine use can cause tachycardia, hypertension, coronary artery spasm with infarction, and pneumothorax resulting in severe, acute chest pain. Exercise, smoking, and family hx of MI can be addressed after the danger of cocaine-related complications has been eliminated.

Normal K+ levels are the same for newborns and older children/adults A. True B. False

B. False -Newborn K+ 3-6 mEq/L Older: 3.5-5.0 mEq/L

The nurse reinforces instructions to mother of a child who has been hospitalized w/croup. Which of following statements by mom would indicate further instruction? a. I will give Tylenol for fever. b. I will give cough syrup every night at bedtime. c. Sips of warm fluids during a croup attack will help. d. I will place a cool-mist humidifier next to my child's bed.

B. I will give my child cough syrup if a cough develops. R: Cough syrups and cold medicines are not to be given, because they may dry and thicken secretions. Adequate hydration of 500 to 1000 mL of fluids daily is imporant for thinning secretions. Acetaminophen is used if a fever develops. Sips of warm fluids during a croup attack help to relax the vocal cords and thin the mucus.

During anti tubercular therapy with isoniazid, the patient received another prescription for pyridoxine. Which statement by the nurse explains the best rationale for this second medication? a. this vitamin will help improve you energy levels b. this vitamin helps to prevent neurologic adverse effects. c. this vitamin works to protect your heart from toxic effects. d. this vitamin helps to reduce gastrointestinal adverse effects.

B. Isoniazid (INH) - typically initial therapy. Effects liver and can cause peripheral neuritis (tingling in fingers & toes) so you give pyridoxine (B6) to prevent. Isoniazid = I = "I don't want to eat". Teach not to eat with medicine. Give either 1 hour before or 2 hours after. No alcohol with this medicine either.

A complete blood workup is ordered for a 5 month old with tetralogy of Fallot. Because of the infant's heart disease, the nurse would expect the report to show: A. Anemia B. Polycythemia C. Agranulocytosis D. Thrombocytopenia

B. Mrs. Reklau emphasized this. The body responds to the chronic hypoxia caused by the heart defect by increasing the production of red blood cells in an attempt to increase the oxygen-carrying capacity of the blood.

A student at a local high school is diagnosed with syphilis. Which of the following actions should the nurse take? A. Report the case to the student's provider B. Report the case to the state health department C. Notify the student's parents D. Notify the school principal

B. Report to CDC. Syphilis reportable in all 50 states, needs to be monitored.

When caring for a 3 year old with tetralogy of Fallot, the nurse expects to see fatigue and poor activity tolerance. This is caused by: A. Poor muscle tone B. Inadequate oxygenation of tissues. C. Restricted blood flow leaving the heart D. Inadequate intake of food.

B. The child's fatigue results from left to right shunting that occurs with tetralogy of Fallot. This shunting causes poorly oxygenated blood to circulate through the body. Poor muscle tone and inadequate food intake can result from this condition, but these are effects, not causes. Restricted blood flow leaving the heart is associated with aortic stenosis.

A clear mucus may indicates that the infection is most likely... A. Bacterial B. Viral C. Fungal

B. Viral

A client who has undergone successful femoral-popliteal bypass grafting of the leg says to the nurse, "I hope everything goes well after this and that I don't lose my leg. I'm so afraid that I'll have gone through this for nothing." The nurse makes which therapeutic response to the client? a) I can understand what you mean. I'd be nervous too, if I were in your shoes b) this surgery is so successful that I wouldn't be concerned at all if I were you c) complications are possible, but you have a good deal of control if you make the lifestyle adjustments we talked about d) stress isn't helpful for you. You should probably just try to relax. You shouldn't worry unless something actually happens.

C - Clients frequently fear that they will ultimately lose a limb or become debilitated in some other way. Option A feeds into the client's anxiety and is not therapeutic. Option B gives false reassurance. Option D is meant to be reassuring, but it offers no suggestions to empower the client. Option C acknowledges the client's concerns and empowers the client to improve his or her health, which will ultimately reduce concern about the risk of complications.

Which of the following physician's order should the nurse question when preparing a client who will undergo coronary artery bypass graft within an hour? a) potassium chloride per slow IV drip b) calcium channel blocker c) digoxin d) prophylactic antibiotic

C - digoxin increases force of cardiac contractility and therefore it increases cardiac workload.

A nurse is caring for a child who has bronchiolitis (RSV). Which of the following are appropriate actions for the nurse to take? a. administer oral prednisone b. initiate chest percussion and postural drainage c. administer humidified oxygen d. suction the nasopharynx as needed e. administer oral penicillin

C and D. Humidified oxygen provides moisture to airway, suctioning the nasopharynx will assist the client to clear recreation and these are appropriate interventions.

What are the drug therapy targets for angina treatment? A. Increase cardiac output; decrease body fluid volume B. Decrease heart rate; increase activation of RAAS system C. Improve myocardial perfusion; reduce metabolic demand

C.

Molly, with suspected rheumatic fever, is admitted to the pediatric unit. When obtaining the child's history, the nurse considers which information to be most important? A A fever that started 3 days ago B Lack of interest in food C A recent episode of pharyngitis D Vomiting for 2 days

C. A recent episode of pharyngitis (strep throat) is the most important factor in establishing the diagnosis of rheumatic fever. Although the child may have a history of fever or vomiting or lack interest in food, these findings are not specific to rheumatic fever.

A 12 year old is diagnosed with hypertension. The nurse understands that hypertension may lead to heart failure. Which assessment finding indicates that the child may have developed heart failure? A. Weight loss B. Bradycardia C. Sudden weight gain D. Bounding peripheral pulses

C. Early signs of heart failure include tachycardia, sudden weight gain, scalp sweating, and weak peripheral pulses. Weight gain can indicate venous congestion. Tachycardia occurs with heart failure as the heart's workload increases. Weak peripheral pulses are a sign of heart failure.

A patient with angina pectoris is being discharged home with nitroglycerine tablets. Which of the following instructions does the nurse include in the teaching? A. "When your chest pain begins, lie down, and place one tablet under your tongue. If the pain continues, take another tablet in 5 minutes." B. "Place one tablet under your tongue. If the pain is not relieved in 15 minutes, go to the hospital." C. "Place one Nitroglycerine tablet under the tongue every five minutes for three doses. Go to the hospital if the pain is unrelieved. D. "Continue your activity, and if the pain does not go away in 10 minutes, begin taking the nitro tablets one every 5 minutes for 15 minutes, then go lie down."

C. Angina pectoris is caused by myocardial ischemia related to decreased coronary blood supply. Giving nitroglycerine will produce coronary vasodilation that improves the coronary blood flow in 3 - 5 mins. If the chest pain is unrelieved, after three tablets, there is a possibility of acute coronary occlusion that requires immediate medical attention.

Which of the following risk factors for coronary artery disease cannot be corrected? A Cigarette smoking B DM C Heredity D HPN

C. Because "heredity" refers to our genetic makeup, it can't be changed. Cigarette smoking cessation is a lifestyle change that involves behavior modification. Diabetes mellitus is a risk factor that can be controlled with diet, exercise, and medication. Altering one's diet, exercise, and medication can correct hypertension.

The most important assessment for the nurse to make after a client has had a femoropopliteal bypass for peripheral vascular disease would be: A Incisional pain B. Pedal pulse rate C. Capillary refill time D Degree of hair growth

C. Checking capillary refill provides data about current perfusion of the extremity. While the presence and quality of the pedal pulse provide data about peripheral circulation, it is not necessary to count the rate.

A nurse is triaging clients in the ED. Which of the following clients should the nurse tell the provider to see first? a. a toddler with asthma and a pulse ox of 95% on 2L b. a toddler with otitis media, fever, and purulent ear discharge c. school aged child with acute epiglottis, is drooling, and has absence of spontaneous cough d. an adolescent who has sickle cell anemia,

C. Epiglottis is a medical emergency

Which medication is usually given to children diagnosed with Kawasaki disease? Discuss A. Acetaminophen (Tylenol) every 4 hours B. Amoxicillin (Amoxil) divided into three daily doses C. Aspirin daily D. Ibuprofen (Motrin) every 6 to 8 hours

C. For kawasaki disease, aspirin is given initially in an anti-inflammatory dose to control fever and symptoms of inflammation. When fever has subsided, aspirin is continued at an antiplatelet dose. If the child develops coronary abnormalities, salicylate therapy is continued indefinitely. Acetaminophen and ibuprofen aren't used because they don't thin the blood. Amoxicillin is an antibiotic, and antibiotics aren't effective in treating kawasaki disease.

Paul was admitted to the ER for chest pain that occurred while he was sleeping and the pain has not resolved. He was diagnosed with Prinzmetal's angina. This form of angina... a. is caused by a blockage of the circumflex artery b. is a benign form of angina c. is usually caused by a coronary vasospasm d. will always resolve with administration of nitroglycerin

C. Primnzmetal's angina is also known as variant angina and is usually caused by a coronary vasospasm. It may occur at rest, long after exercise, or during sleep.

An 8 year old is admitted with myocarditis and associated tachycardia, and is prescribed fuosemide (Lasix). Which lab value does the nurse need to closely monitor for this child? A. Calcium B. Glucose C. Potassium D. Sodium

C. potassium

Exceeding which of the following serum cholesterol levels significantly increases the risk of coronary artery disease? A 100 mg/dl B 150 mg/dl C 175 mg/dl D 200 mg/dl

Cholesterol levels above 200 mg/dl are considered excessive. They require dietary restriction and perhaps medication. Exercise also helps reduce cholesterol levels. The other levels listed are all below the nationally accepted levels for cholesterol and carry a lesser risk for CAD.

The nurse is caring for a 9-month-old who was born with a congenital heart defect (CHD). Assessment reveals a HR of 160, capillary refill of 4 seconds, bilateral crackles, and sweat on the scalp. These are signs of _____________________.

Congestive heart failure or CHF.

Croup symptoms may include: Ronchi, Rales and a "barky" or "bark like" __________

Cough

A nurse is carting for a child who recently underwent a cardiac catheterization to diagnose a congenital heart defect. Which finding indicates the need for immediate action? Discuss A. Increased Pulse B. Decreased urine output C. Increased temperature D. Bleeding from the catheter site.

D. Bleeding from the catheter site may become life threatening and demands immediate action. Immediately apply pressure to the site. An increased pulse indicates pain and the need for medication, which the nurse should give if other signs of pain are present, but it isn't an emergency intervention. Because a child must remain flat after a cardiac cath, a decrease in urine output may occur, but it doesn't require immediate action unless urine output is absent. An increased body temperature after cardiac catheterization is not abnormal.

Exceeding which of the following serum cholesterol levels significantly increases the risk of coronary artery disease? A 100 mg/dl B 150 mg/dl C 175 mg/dl D 200 mg/dl

D. Cholesterol levels above 200 mg/dl are considered excessive. They require dietary restriction and perhaps medication. Exercise also helps reduce cholesterol levels. The other levels listed are all below the nationally accepted levels for cholesterol and carry a lesser risk for CAD.

Which of the following types of pain is most characteristic of angina? A Knifelike B Sharp C Shooting D Tightness

D. The pain of angina usually ranges from a vague feeling of tightness to heavy, intense pain. Pain impulses originate in the most visceral muscles and may move to such areas as the chest, neck, and arms.

A client is scheduled for a cardiac catheterization using a radiopaque dye. Which of the following assessments is most critical before the procedure? A Intake and output B Baseline peripheral pulse rates C Height and weight D Allergy to iodine or shellfish

D. This procedure requires an informed consent because it involves injection of a radiopaque dye into the blood vessel. The risk of allergic reaction and possible anaphylaxis is serious and must be assessed before the procedure.

A patient experienced sudden cardiac death (SCD) and survived. What should the nurse expect to be used as preventive treatment for the patient? A. External pacemaker B. An electrophysiologic study (EPS) C. Medications to prevent dysrhythmias D. Implantable cardioverter-defibrillator (ICD)

D. Implantable cardioverter-defibrillator (ICD) An ICD is the most common approach to preventing recurrence of SCD. An external pacemaker may be used in the hospital but will not be used for the patient living daily life at home. An EPS may be done to determine if a recurrence is likely and determine the most effective medication treatment. Medications to prevent dysrhythmias are used but are not the best prevention of SCD.

A child is suspected of having Kawasaki disease. Which finding is significant? A. Extreme lethargy B. Increased appetite C. Respiratory congestion D. Fever for at least 5 days

D. Kawasaki disease is a type of vasculitis affecting small to medium sized vessels. It primarily affects the lymph nodes but may progress to the coronary arteries. A child with Kawasaki disease has afever for at least five days along with an erythematous rash, red tongue, and red, cracked dry lips. Irritability, not lethargy is seen in Kawasaki disease, along with decreased appetite and edema of the hands and feet. Respiratory congestion isn't a common symptom.

A nurse is teaching a group of parent about influenza. Which of the following should be included in the teaching? a. amantadine will prevent the illness b. rimantadine is administered intramuscularly c. zanamivir can be given to children 1 year and older d. oseltamivir should be given within 48 hours of onset of symptoms

D. Oseltamivir is Tamiflu - decreases severity of flu manifestations

A nurse is admitting an infant that has RSV. Which of the following rooms should the nurse assign the client? A. a semi-private room with a client that has croup b. a semi-private room with a client that has pneumonia c. a private room with droplet/contact precautions d. a private room with protective isolation

D. RSV requires contact and droplet precautions. The client either needs a private room, or a semi-private room with another client with RSV. **RSV contact precautions for this test

Which nursing intervention best helps decrease anxiety for the parents of a child scheduled for cardiac surgery? Discuss A. Tell the parents not to worry, because the physician performs this procedure all the time. B. Obtain an order for anti-anxiety medication for the parents, if requested. C. Teach the parents and the child about the surgery 1 month before the procedure D. Explain the steps that will occur before and after surgery. The parents need something tangible to focus on.

D. Telling the parents about the sequence of events before and after surgery will decrease their anxiety and increase cooperation. The nurse should listen to the parents' concerns, rather than dismissing them by telling them not to worry. It isn't appropriate to obtain an order for anti-anxiety medication for the parents. Children do best with preoperative teaching 3 to 7 days before a procedure rather than 1 month before.

ECHO can be used to detect the pressure in the chambers of the heart and it's surrounding vessels. A. True B. False

FALSE.Cardiac Catheterization is used for pressure readings inside the heart.

If epiglottitis is suspected, the nurse should attempt to visualize the epiglottis without the use of tongue depressor. A. True B. False

False - while the tongue depressor shouldn't be used, Only MD should attempt to visualize the epiglottis while intubating, not nurse (suspected diagnosis is enough diagnosis). Nurse need to have emergency ETT/Tradch set at bedside. There will likely be Lateral neck film in ER

What is the mechanism by which organic nitrates exert their effects? A. Increased PKG which causes relaxation of large veins and arteries B. Increased PKG which causes relaxation of smooth muscle C. Decreased PKG which causes relaxation of smooth muscle

Increased PKG which causes relaxation of smooth muscle

A child who has reddened eyes with no discharge; red, swollen, and peeling palms and soles of the feet; dry, cracked lips; and a "strawberry tongue" most likely has _____________________

Kawasaki disease or KD

Patent ductus arteriosus causes what type of shunt? _____________________

Left to right. Blood flows from the higher pressure aorta to the lower pressure pulmonary artery, resulting in a left to right shunt

The flow of blood through the heart with an atrial septal defect (ASD) is _____________________.

Left to right. The pressures in the left side of the heart are greater, causing the flow of blood to be from an area of higher pressure to lower pressure, or left to right, increasing the pulmonary blood flow with the

A mother arrives at the hospital emergency department with her child, in whom a diagnosis of epiglottitis is documented. Which prescription, if written by the health care provider, should the nurse question? 1. Obtain a throat culture. 2. Obtain axillary temperatures. 3. Administer humidified oxygen. 4. Administer acetaminophen (Tylenol) for fever.

Obtain a throat culture

For the child with coarctation of aorta, which drug may be given to allow the patent ductus arteriosus (PDA) to remain open until surgery? _____________________

Prostaglandin E Also given to keep PDA open in hypo plastic left heart syndrome and tricuspid atresia

The right forearm of a client who had a purified protein derivative (PPD) test for tuberculosis is reddened and raised about 3mm where the test was given. This PPD would be read as having which of the following results? A Indeterminate B Needs to be redone C Negative D Positive

Question 9 Explanation: C This test would be classed as negative. A 5 mm raised area would be a positive result if a client was HIV+ or had recent close contact with someone diagnosed with TB. Indeterminate isn't a term used to describe results of a PPD test. If the PPD is reddened and raised 10mm or more, it's considered positive according to the CDC.

Following an earthquake, patients are triaged by emergency medical personnel and are transported to the hospital. Which of these patient will the nurse need to assess first? a. b. c. d.

Red tag - indicates emergent/life threatening

A 10-year-old child is recovering from a severe sore throat. The parent states that the child complains of chest pain. The nurse observes that the child has swollen joints, nodules on the fingers, and a rash on the chest. The likely cause is _____________________.

Rheumatic fever or RF. To make the diagnosis of RF, major and minor criteria are used. Major criteria include carditis, subcutaneous nodules, erythema marginatum, chorea, and arthritis. Minor criteria include fever and previous history of RF.

Decreased urinary output (fewer wet diapers or less frequent toileting) may be a sign of worsening heart failure. A. True B. False

TRUE

Some structural differences of the airway in children compared to adults are: Larynx and glottis are located higher in the neck, tonsils are larger, and some cartilage is less developed. A. True B. False

TRUE

Digoxin should not be administered to a child (1 year and older) with a heart rate of less than 70. A. True B. False

TRUE - again, dr will have set heart rate and write order to hold below that.

A 6-month-old who has episodes of cyanosis after crying could have the congenital heart defect (CHD) of decreased pulmonary blood flow called _____________________.

Tetralogy of Fallot or TOF which is a *decreased pulmonary blood flow classification of heart defects. **No meds fix TofF, must have surgery

While assessing a newborn with respiratory distress, the nurse auscultates a machine-like heart murmur. Other findings are a wide pulse pressure, periods of apnea, increased PaCO2, and decreased PO2. The nurse suspects that the newborn has: 1. Pulmonary hypertension. 2. Patent ductus arterioles (PDA) 3. Ventrical septal defect (VSD) 4. Bronchopulmonary displasia

The main identifier in the stem is the machine-like murmur, which is the hallmark of a PDA.

Which of the following blood tests is most indicative of cardiac damage? A Lactate dehydrogenase B Complete blood count (CBC) C. Troponin I D Creatine kinase (CK)

Troponin I levels rise rapidly and are detectable within 1 hour of myocardial injury. Troponin levels aren't detectable in people without cardiac injury.

Epiglottitis is almost always caused by Haemophilus influenza 1. True 2. False

True **HiB vaccine recommended for prevention

Charge nurse is planning room assignment for a 24 yr old coming from ER. Report states production cough, a suspicious chest xray, a positive Mantoux test, and pending acid fast bacillus sputum test. What type of room should the nurse place the client?

Until proven otherwise, the client is suspected of having TB and should be placed in a private, respiratory isolation room with negative pressure.

What are the three kinds of temporary pacing?

What are the three kinds of temporary pacing? 1. Transcutaneous. 2. Transvenous. 3. Epicardial.

Which of the following conditions would decrease after load? a. use of nitrates b. hypothermia c. aortic stenosis d. hypertension

a. Afterload is the resistance the heart must pump against. Nitrates are vasodilators, so their use decreases resistance.

Matthew was started on esmolol for severe hypertension refractive to other drug regimens. Esmolol is incompatible to which of the following medications? A. cimetidine b. furosemide c. penicillin G potassium d. midazolam

b. furosemide - These are incompatible for the same venous access port if both being given IV. Another IV may be started, or a PO form may be used. However, using these meds together may lower BP and slow heart rate. Watch for dizziness, weakness, fainting, and irregular heart beats.

A child is admitted to the hospital with a diagnosis of rheumatic fever. The nurse reviews the blood laboratory findings, knowing that which of the following will confirm the likelihood of this disorder? a. increased leukocyte count b. decreased hemoglobin count c. increased antistreptolysin-O (ASO) d. decreased erythrocyte sedimentation rate

c. increased antistreptolysin-O (ASO) Children suspected of having rheumatic fever are tested for streptococcal antibodies. The most reliable and best standardized test to confirm the diagnosis is the ASO titer. An elevated level indicates the presence of rheumatic fever.

What are some common pacemaker malfunctions?

1. Failure to Pace 2. Failure to Capture 3. Failure to Sense (undersensing) 4. Oversensing

The nurse is caring for a child with Kawasaki disease (KD). A student nurse who is on the unit asks if there are medications to treat this disease. The nurse's response to the student nurse is: 1. Immunoglobulin G and aspirin. 2. Immunoglobulin G and ACE inhibitors. 3. Immunoglobulin E and heparin 4. Immunoglobulin E and ibuprofen

1. High-dose immunoglobulin G (??IVIG) and salicylate therapy for inflammation are the current treatment for KD. The immunoglobulin is a one time dose, it runs per 8-12 hours, needs to be given within 10 days of symptoms (the earlier, the better). If fever doesn't go down within 36 hours or if it goes down then spikes back up, can give one more dose.

Which physiological changes occur as a result of hypoxemia in congestive heart failure (CHF)? 1. Polycythemia and clubbing. 2. Anemia and barrel chest. 3. Increased white blood cells and low platelets. 4. Elevated erythrocyte sedimentation rate and peripheral edema.

1. Polycythemia and clubbing. The hypoxemia stimulates erythropoiesis, which causes polycythemia, in an attempt to increase oxygen by having more red blood cells carry oxygen. Clubbing of the fingers is a result of the polycythemia and hypoxemia.

A child diagnosed with congestive heart failure (CHF) is receiving maintenance doses of digoxin and furosemide. She is rubbing her eyes when she is looking at the lights in the room, and her HR is 70 beats per minute. The nurse expects which laboratory finding? 1. Hypokalemia. 2. Hypomagnesemia 3. Hypocalcemia 4. Hypophosphatemia

1. The rubbing of the child's eyes may mean that she is seeing halos around the lights, indicating digoxin toxicity. The HR is slow for her age and also indicates digoxin toxicity. A decrease in serum potassium because of the furosemide can increase the risk for digoxin toxicity

The school nurse has been following a child who comes to the office frequently for vague complaints of dizziness and headache. Today, she is brought in after fainting in the cafeteria following a nosebleed. Her BP is 122/85, and her radial pulses are bounding. The nurse suspects she has: 1. Transposition of the great vessels. 2. Coarctation of the aorta (COA). 3. Aortic stenosis (AS). 4. Pulmonic stenosis (PS).

2. In the older child, COA causes dizziness, headache, fainting, elevated blood pressure, and bounding radial pulses

Aspirin has been ordered for the child with rheumatic fever (RF) in order to: 1. Keep the patent ductus arteriosus (PDA) open. 2. Reduce joint inflammation. 3. Decrease swelling of strawberry tongue. 4. Treat ventricular hypertrophy of endocarditis

2. Joint inflammation is experienced in RF; aspirin therapy helps with inflammation and pain.

Nurse caring for a hospitalized infant w/bronchiolitis. Diagnostic tests have confirmed respiratory syncytial virus (RSV). On basis of this finding, which w/be most appropriate nursing action? 1. Initiate strict enteric precautions. 2. Move the infant to a room with another child with RSV. 3. Leave the infant in the present room because RSV is not contagious. 4. Inform the staff that they must wear a mask, gloves, and a gown when caring for the child.

2. Plan to move the infant to a room w/another child w/RSV. R: RSV is a highly communicable disorder, but it is not transmitted via airborne route. It is usually transferred by hands, and meticulous handwashing is necesary to decrease spread of organisms. The infant w/RSV is isolated in a single room or placed in a room w/another child w/RSV. Nurse s/wear a gown when the soiling of clothing may occur.

While looking through the chart of an infant with a congenital heart defect (CHD) of decreased pulmonary blood flow, the nurse would expect which laboratory finding? 1. Decreased platelet count. 2. Polycythemia. 3. Decreased ferritin level. 4. Shift to the left.

2. Polycythemia is the result of the body attempting to increase the oxygen supply in the presence of hypoxia by increasing the total number of red blood cells to carry the oxygen.

The nurse is caring for an 8-year-old girl whose parents indicate she has developed spastic movements of her extremities and trunk, facial grimace, and speech distur- bances. They state it seems worse when she is anxious and does not occur while sleeping. The nurse questions the parents about which recent illness? 1. Kawasaki disease (KD). 2. Strep throat. 3. Malignant hypertension. 4. Atrial fibrillation.

2. Strep throat. Choryea can be a manifestation of RF, with a higher incidence in females

A mother arrives at the clinic with her 3-year-old child. The mother tells the nurse that the child has had a fever and a cough for the past 2 days and that this morning the child began to wheeze. Viral pneumonia is diagnosed. Based on the diagnosis, the nurse anticipates that which will be a component of the treatment plan? 1. Oral antibiotics 2. Supportive treatment 3. IV fluid administration 4. Hospitalization and intravenous (IV) antibiotics

2. Supportive treatment Viral antibiotics = no antibiotics.

The Norwood procedure is used to correct: 1. Transposition of the great vessels. 2. Hypoplastic left heart syndrome. 3. Tetralogy of Fallot (TOF). 4. Patent ductus arteriosus (PDA).

2. The Norwood procedure is specific to hypoplastic left heart syndrome

The parents of a 3-month-old ask why their baby will not have an operation to correct a ventricular septal defect (VSD). The nurse's best response is: 1. "It is always helpful to get a second opinion about any serious condition like this." 2. "Your baby's defect is small and will likely close on its own by one year of age 3. It is common for physicians to wait until infants experience respiratory distress before they perform the surgery 4. With a small defect like this, it is common to wait until the child is 10 years of age

2. Usually a VSD will close on its own within the first year of life.

Family teaching has been effective when the parent of a toddler diagnosed with Kawasaki disease (KD) states: 1. "The arthritis in her knees is permanent. She will need knee replacements." 2. "I will giver her diphenhydramine (Benadryl) for her peeling palms and soles of her feet." 3. "I know she will be irritable for 2 months after her symptoms started. " 4. "I will continue with high doses of Tylenol for her inflammation."

3. Children can be irritable for 2 months after the symptoms of the disease start

During a well-child checkup for an infant with tetralogy of Fallot (TOF), the child develops severe respiratory distress and becomes cyanotic. The nurse's first action should be to: 1. Lay the child flat to promote hemostasis. 2. Lay the child flat with legs elevated to increase blood flow to the heart 3. Sit the child on the parents lap with legs dangling, to promote venous pooling 4. Hold the child in knee-chest position to decrease venous blood return

4. The increase in the SVR would increase afterload and increase blood return to the pulmonary artery. This choice also decreases the preload in this client. Knee to chest position if you witness the tet spell. If you did not witness the spell, assess child first.

A child has been diagnosed with valvular disease following rheumatic fever (RF). During patient teaching, the nurse discusses the child's long-term prophylactic therapy with antibiotics for dental procedures, surgery, and childbirth. The parents indicate they understand when they say: 1. "She will need to take the antibiotics until she is 18 years of age 2. She will need to take the antibiotics for 5 years following the last attack 3. She will need to take the antibiotics for 10 years after the last attack 4. She will need to take the antibiotics for the rest of her life.

4. Valvular involvement indicates significant damage, so antibiotics would be taken for the rest of her life. Prophylactically with any dental/invasive procedures

A nine year old received digoxin (Lanoxin) daily for the past 5 days of his hospitalization. Before giving him his dose this morning, the nurse performs a routine assessment. Which assessment finding indicates the need to hold the child's morning dose of digoxin? A. Vomiting B. Palpitations C. Increased heart rate D. Serum digoxin level of 1.2 ng/mL

A -Vomiting is a sign of digoxin toxicity. Palpitations and increased heart rate indicate that digoxin is needed. The serum level is within the normal range

A nurse is caring for a nine year old experiencing tachycardia due to myocarditis. Digoxin (lanoxin) is prescribed. Before giving digoxin to this child, the nurse should assess: A. Apical pulse B. Urine output C. Radial pulse D. Blood pressure

A -digoxin slows the heart rate and strengthens contractions; it shouldn't be given if the heart rate is abnormally low with regard to the child's age. The most accurate measure of the child's heart rate is the apical (not radial) pulse. Urine output and blood pressure don't need to be assessed before digoxin administration. The physician will write an ordered rate to hold the dig

Which of the following Medicines are commonly not used in treating children's respiratory symptoms? (choose all that apply) A. Antitussives B. Antibiotics C. Decongestants

A and C (not common in under 6, even then use cautiously

When providing nutritional counseling for patients at risk for CAD, which foods would the nurse encourage patients to include in their diet (select all that apply)? A. Tofu B. Walnuts C. Tuna fish D. Whole milk E. Orange juice

A, B, C. Tuna fish, tofu, and walnuts are all rich in omega-3 fatty acids, which have been shown to reduce the risks associated with CAD when consumed regularly.

A nurse is assessing a child. Which of the following are clinical manifestations of epiglottitits? Select all a. hoarseness and difficulty speaking b. difficulty swallowing c. low-grade fever d. drooling e. dry, barking cough f. stridor

A, B, D, F (high grade fever would be, not low.)

Which of the following are correct statements regarding Digoxin (Lanoxin)? Check all that apply. A. Digoxin is the drug of choice to improve myocardial contractility B. Often prescribed to increase contractility and decrease afterload C. ALWAYS check dose with another Registered Nurse before administration D. Administration is normally IV for infants

A, B, and C. Administer orally to the side of the mouth and rinse with water. Do not give more than 1ml/0.5mg in one dose to an infant.

Which interventions decrease cardiac demands in an infant with congestive heart failure (CHF)? Select all that apply. A. Allow parents to hold and rock their child. B. Feed only when the infant is crying. C. Keep the child uncovered to promote low body temperature. D. Make frequent position changes E. Feed the child when sucking his fists F. Change bed linens only when necessary G. Organize nursing activities

A, D, E, F, G - 1. Rocking by the parents will comfort the infant and decrease demands. 4. Frequent position changes will decrease the risk for infection by avoiding immobility with its potential for skin breakdown. 5. An infant sucking the fists could indicate hunger. 6. Change bed linens only when necessary to avoid disturbing the child. 7. Organize nursing activities to avoid disturbing the child.

When teaching a client about propranolol hydrochloride, the nurse should base the information on the knowledge that propranolol hydrochloride: A Blocks beta-adrenergic stimulation and thus causes decreased heart rate, myocardial contractility, and conduction. B Increases norepinephrine secretion and thus decreases blood pressure and heart rate. C Is a potent arterial and venous vasodilator that reduces peripheral vascular resistance and lowers blood pressure. D Is an angiotensin-converting enzyme inhibitor that reduces blood pressure by blocking the conversion of angiotensin I to angiotensin II

A- propranolol hydrochloride is a beta-adrenergic blocking agent. Actions of propranolol hydrochloride include reducing heart rate, decreasing myocardial contractility, and slowing conduction.

The nurse is aware that a common physiologic adaptation of children with tetralogy of Fallot is: A. Clubbing of fingers B. Slow, irregular respirations C. Subcutaneous hemorrhages D. Decreased red blood cell count

A. Hypoxia leads to poor peripheral circulation; clubbing occurs as a result of additional capillary development and tissue hypertrophy of the fingertips.

Following a treadmill test and cardiac catheterization, the client is found to have coronary artery disease, which is inoperative. He is referred to the cardiac rehabilitation unit. During his first visit to the unit he says that he doesn't understand why he needs to be there because there is nothing that can be done to make him better. The best nursing response is: A "Cardiac rehabilitation is not a cure but can help restore you to many of your former activities." B "Here we teach you to gradually change your lifestyle to accommodate your heart disease." C "You are probably right but we can gradually increase your activities so that you can live a more active life." D "Do you feel that you will have to make some changes in your life now?"

A. Such a response does not have false hope to the client but is positive and realistic. The answer tells the client what cardiac rehabilitation is and does not dwell upon his negativity about it.

A child is in the pediatric intensive care unit immediately after cardiac surgery. Which nursing action is most important? A. Assess the airway. B. Administer sedation C. Maintain semi-Fowler's position. D. Monitor oxygen saturation readings.

A. Surprise!!!! Child will return from surgery with ET tube and nurse should check for bilateral breath sounds to evaluate tube placement. Just a thought. What would we do if two of the choices were 1.) assess airway & 2.) perform hand hygiene? My head would explode.

A child underwent cardiac surgery and the nurse must prepare his parents for discharge. Which discharge instruction is correct? A. "Call your doctor before your child has dental care." B. "Keep your child away from other children for 6 months." C. "if your child vomits his digoxin, he may need a second dose". D. "Encourage the child to participate in activities so he can develop normally."

A. Upon discharge, parents should be taught to call the physician before the child has dental care. The child may be at risk for bacterial endocarditis after surgery, and dental procedures are a common portal of entry for bacteria. The physician may order antibiotics before a dental procedure.

Mr. Cole was admitted to ER with cough, fever, chills, anorexia, malaise, and headache. He currently has a pericardial friction rub and has a history of rheumatic fever. While examining Mr. Cole, you note fine, dark lines in his nail beds and some flat lesions on his palms. These flat lesions are known as a. Janeway lesions b. Osler's nodes c. Roth spots d. Pella's sign

A. Mr. Cole has endocardidits - Laneway lesions are flat and painless erythematous areas found on palms and soles of feet predominantly. Osler's nodes are small painful nodules also associated with endocarditis and found on fingers and toes.

The community health nurse is planning health promotion teaching targeted at preventing coronary artery disease (CAD). Which ethnic group would the nurse select as the highest priority for this intervention? A. White male B. Hispanic male C. African American male D. Native American female

A. White male The incidence of CAD and myocardial infarction (MI) is highest among white, middle-aged men. Hispanic individuals have lower rates of CAD than non-Hispanic whites or African Americans. African Americans have an earlier age of onset and more severe CAD than whites and more than twice the mortality rate of whites of the same age. Native Americans have increased mortality in less than 35-year-olds and have major modifiable risk factors such as diabetes.

A 52-year-old male patient has received a bolus dose and an infusion of alteplase (Activase) for an ST-segment elevation myocardial infarction (STEMI). To determine the effectiveness of this medication, the nurse should assess the patient for the A. presence of chest pain. B. blood in the urine or stool. C. tachycardia with hypotension. D. decreased level of consciousness.

A. presence of chest pain. Alteplase is a fibrinolytic that is administered to patients who have had an STEMI. If the medication is effective, the patient's chest pain will resolve because the medication dissolves the thrombus in the coronary artery and results in reperfusion of the myocardium. Bleeding is a major complication of fibrinolytic therapy. Signs of major bleeding include decreased level of consciousness, blood in the urine or stool, and increased heart rate with decreased blood pressure.

The nurse teaches the client with angina about the common expected side effects of nitroglycerin, including: A Headache B High blood pressure C Shortness of breath D Stomach cramps

A.Because of the widespread vasodilating effects, nitroglycerin often produces such side effects as headache, hypotension, and dizziness. The client should lie or sit down to avoid fainting. Nitro does not cause shortness of breath or stomach cramps.

Which of the following symptoms should the nurse teach the client with unstable angina to report immediately to her physician? A A change in the pattern of her pain B Pain during sex C Pain during an argument with her husband D Pain during or after an activity such as lawn mowing

A.The client should report a change in the pattern of chest pain. It may indicate increasing severity of CAD.

Cardiac magnetic resonance imaging (MRI) is prescribed for a client. The nurse identifies that which of the following is a contraindication for performance of this diagnostic study? A) Client has a pacemaker B) Client is allergic to iodine C) Client has diabetes mellitus D) Client has a biological porcine valve

Ans: A The magnetic fields used for magnetic resonance imaging (MRI) can deactivate the pacemaker. Options 2, 3, and 4 are not contraindications for an MRI.

A client with coronary artery disease is scheduled to have a diagnostic exercise stress test. The nurse plans to include which of the following items in client teaching about this procedure? A) Avoid cigarettes for 30 minutes before the procedure B) Wear loose clothing with a shirt that buttons in front C) Eat breakfast just before the procedure D) Wear firm, rigid shoes such as workboots

Ans: B The client should wear loose, comfortable clothing for the procedure. Electrocardiogram (ECG) lead placement is enhanced if the client wears a shirt that buttons in the front. The client should wear rubber-soled, supportive shoes, such as athletic training shoes. The client should receive nothing by mouth after bedtime, or for a minimum of 2 hours before the test. The client should avoid smoking, alcohol, and caffeine on the day of the test. Inadequate or incorrect preparation can interfere with the test, with the potential for a false-positive result.

A client with angina complains that the anginal pain is prolonged and severe and occurs at the same time each day, most often at rest in the absence of precipitating factors. How would the nurse best describe this type of anginal pain? A) Stable angina B) Variant angina C) Unstable angina D) Nonanginal pain

Ans: B Variant angina, or Prinzmetal's angina, is prolonged and severe and occurs at the same time each day, most often at rest. Stable angina is induced by exercise and relieved by rest or nitroglycerin tablets. Unstable angina occurs at lower and lower levels of activity or at rest, is less predictable, and is often a precursor of myocardial infarction

A client is in sinus bradycardia with a heart rate of 45 beats/min, complains of dizziness, and has a blood pressure of 82/60 mm Hg. Which of the following should the nurse anticipate will be prescribed? A). Defibrillate the client B). Administer digoxin (Lanoxin) C). Continue to monitor the client D). Prepare the transcutaneous pacing

Ans: D Hypotension and dizziness are signs of decreased cardiac output. Transcutaneous pacing provides a temporary measure to increase the heart rate and thus perfusion in the symptomatic client. Digoxin will further decrease the client's heart rate. Defibrillation is used for treatment of pulseless ventricular tachycardia and ventricular fibrillation. Continuing to monitor the client delays necessary intervention.

The charge nurse on the cardiac unit is planning assignments for the day. Which of the following is the most appropriate assignment for the float nurse that has been reassigned from labor and delivery? A. A one-week postoperative coronary bypass patient, who is being evaluated for placement of a pacemaker prior to discharge. B. A suspected myocardial infarction patient on telemetry, just admitted from the Emergency Department and scheduled for an angiogram. C. A patient with unstable angina being closely monitored for pain and medication titration. D. A post-operative valve replacement patient who was recently admitted to the unit because all surgical beds were filled.

Answer: A The charge nurse planning assignments must consider the skills of the staff and the needs of the patients. The labor and delivery nurse who is not experienced with the needs of cardiac patients should be assigned to those with the least acute needs. The patient who is one-week post-operative and nearing discharge is likely to require routine care. A new patient admitted with suspected MI and scheduled for angiography would require continuous assessment as well as coordination of care that is best carried out by experienced staff. The unstable patient requires staff that can immediately identify symptoms and respond appropriately. A post-operative patient also requires close monitoring and cardiac experience.

A newborn is diagnosed with coarctation of the aorta. The infant is discharged with a prescription for digoxin (lanoxin) 0.05 mg PO every 12 hours. The bottle of digoxin is labeled 0.15 mg in 1/2 teaspoon, the nurse should teach the mother to administer the medication using a: A. Nipple B. Calibrated syringe C. Plastic measuring spoon D. Bottle with an ounce of water

B -A calibrated syringe or dropper provides the most accurate measurement of the medication.

A home health nurse is teaching a client who has active TB. The provider has prescribed the following medication regimen; INH 250mg PO daily, RIF 500mg PO daily, PZA 750mg PO daily, and ethambutol 1 mg PO daily. Which of the following client statements indicate an understanding of the teaching? SATA a. I can substitute one medication for another if I run out because they all fight infection b. i will wash my hands each time I cough c. I will wear a mask when I am in a public are d. I am glad I don't have to have any more sputum specimens e. I don't need to worry where I go once I start taking my medications

B and C. Wash hands and wear mask to prevent spread.

Factors that increase the risk/frequency of respiratory infections in children include: (choose all that apply) A. Consumption of canned food B. Smaller airway diameter C. Shorter distances between structures of the airway D. Short eustachian tubes E. Repeated exposure to "Baby Mozart"

B, C, D - shorter/smaller respiratory system

The nurse would assess a patient with complaints of chest pain for which clinical manifestations associated with a myocardial infarction (MI) (select all that apply)? A. Flushing B. Ashen skin C. Diaphoresis D. Nausea and vomiting E. S3 or S4 heart sounds

B, C, D, E. During the initial phase of an MI, catecholamines are released from the ischemic myocardial cells, causing increased sympathetic nervous system (SNS) stimulation. This results in the release of glycogen, diaphoresis, and vasoconstriction of peripheral blood vessels. The patient's skin may be ashen, cool, and clammy (not flushed) as a result of this response. Nausea and vomiting may result from reflex stimulation of the vomiting center by severe pain. Ventricular dysfunction resulting from the MI may lead to the presence of the abnormal S3 and S4 heart sounds.

The nurse is monitoring for liver toxicity in a patient who has been taking long term INH. Manifestations of liver toxicity include: (SATA) a. orange discoloration of sweat and tears b. darkened urine c. dizziness d. fatigue e. jaundice f. visual disturbances

B, D, E

Which antilipemic medications should the nurse question for a patient with cirrhosis of the liver (select all that apply)? A. Niacin (Nicobid) B. Ezetimibe (Zetia) C. Gemfibrozil (Lopid) D. Atorvastatin (Lipitor) E. Cholestyramine (Questran)

B, D. Ezetimibe (Zetia) should not be used by patients with liver impairment. Adverse effects of atorvastatin (Lipitor), a statin drug, include liver damage and myopathy. Liver enzymes must be monitored frequently and the medication stopped if these enzymes increase. Niacin's side effects subside with time, although decreased liver function may occur with high doses. Cholestyramine is safe for long-term use.

After teaching a patient with chronic stable angina about nitroglycerin, the nurse recognizes the need for further teaching when the patient makes which statement? A. "I will replace my nitroglycerin supply every 6 months." B. "I can take up to five tablets every 3 minutes for relief of my chest pain." C. "I will take acetaminophen (Tylenol) to treat the headache caused by nitroglycerin." D. "I will take the nitroglycerin 10 minutes before planned activity that usually causes chest pain."

B. "I can take up to five tablets every 3 minutes for relief of my chest pain." The recommended dose of nitroglycerin is one tablet taken sublingually (SL) or one metered spray for symptoms of angina. If symptoms are unchanged or worse after 5 minutes, the patient should be instructed to activate the emergency medical services (EMS) system. If symptoms are improved, repeat the nitroglycerin every 5 minutes for a maximum of three doses and contact EMS if symptoms have not resolved completely.

What is the first intervention for a client experiencing myocardial infarction? A. Administer morphine B. Administer oxygen C Administer sublingual nitroglycerin D Obtain an electrocardiogram

B. (But in our notes, she lists it as A ON M with aspirin before oxygen) Administering supplemental oxygen to the client is the first priority of care. The myocardium is deprived of oxygen during an infarction, so additional oxygen is administered to assist in oxygenation and prevent further damage. Morphine and sublingual nitroglycerin are also used to treat MI, but they're more commonly administered after the oxygen. An ECG is the most common diagnostic tool used to evaluate MI.

Which patient is at greatest risk for sudden cardiac death? A. A 42-year-old white woman with hypertension and dyslipidemia B. A 52-year-old African American man with left ventricular failure C. A 62-year-old obese man with diabetes mellitus and high cholesterol D. A 72-year-old Native American woman with a family history of heart disease

B. A 52-year-old African American man with left ventricular failure Patients with left ventricular dysfunction (ejection fraction < 30%) and ventricular dysrhythmias after MI are at greatest risk for sudden cardiac death (SCD). Other risk factors for SCD include (1) male gender (especially African American men), (2) family history of premature atherosclerosis, (3) tobacco use, (4) diabetes mellitus, (5) hypercholesterolemia, (6) hypertension, and (7) cardiomyopathy.

When palpating the brachial, radial, and femoral pulses of a neonate, the nurse notes a difference in pulse amplitude between the femoral and radial pulses bilaterally. This difference suggests: A. Patent ductus arteriosus B. Coarctation of the aorta C. Diminished cardiac output D. Left to right shunting in the heart.

B. A difference in pulse amplitude between the upper and lower extremities or between the femoral and radial pulses suggests a coarctation of the aorta (narrowing of the aorta below the left subclavian artery). A patent ductus arteriousus is associated with a bounding pulse due to left-to-right shunting of blood in the heart. A weak or thinner pulse indicates diminished cardiac output.

A nurse is teaching a client with TB. Which of the following statements should nurse include in teaching? a. you will need to continue to take the multi medication regimen for 4 months b. you will need to provide sputum samples every 4 weeks to monitor the effectiveness of the medication. c. you will need to remain hospitalized for treatment d. you will need to wear a mask at all times

B. Acid fast bacillus sputum tests performed every 2-4 weeks

The patient comes to the ED with severe, prolonged angina that is not immediately reversible. The nurse knows that if the patient once had angina related to a stable atherosclerotic plaque and the plaque ruptures, there may be occlusion of a coronary vessel and this type of pain. How will the nurse document this situation related to pathophysiology, presentation, diagnosis, prognosis, and interventions for this disorder? A. Unstable angina Incorrect B. Acute coronary syndrome (ACS) C. ST-segment-elevation myocardial infarction (STEMI) D. Non-ST-segment-elevation myocardial infarction (NSTEMI)

B. Acute coronary syndrome (ACS) The pain with ACS is severe, prolonged, and not easy to relieve. ACS is associated with deterioration of a once-stable atherosclerotic plaque that ruptures, exposes the intima to blood, and stimulates platelet aggregation and local vasoconstriction with thrombus formation. The unstable lesion, if partially occlusive, will be manifest as unstable angina or NSTEMI. If there is total occlusion, it is manifest as a STEMI.

Atherosclerosis impedes coronary blood flow by which of the following mechanisms? A Plaques obstruct the vein B Plaques obstruct the artery C Blood clots form outside the vessel wall D Hardened vessels dilate to allow blood to flow through

B. Arteries, not veins, supply the coronary arteries with oxygen and other nutrients. Atherosclerosis is a direct result of plaque formation in the artery. Hardened vessels can't dilate properly and, therefore, constrict blood flow.

Atherosclerosis impedes coronary blood flow by which of the following mechanisms? A Plaques obstruct the vein B Plaques obstruct the artery C Blood clots form outside the vessel wall D Hardened vessels dilate to allow the blood to flow through

B. Arteries, not veins, supply the coronary arteries with oxygen and other nutrients. Atherosclerosis is a direct result of plaque formation in the artery. Hardened vessels can't dilate properly and, therefore, constrict blood flow.

What causes unstable angina? A. Atherosclerosis which reduces O2 to tissue B. Atherosclerosis which ruptures, thrombus forms on top, often leading to complete occlusion C. Atherosclerosis which ruptures, thrombus forms on top, always followed by breaking up and embolus formation leading to complete occlusion and infarction

B. Atherosclerosis which ruptures, thrombus forms on top, often leading to complete occlusion

The physician refers the client with unstable angina for a cardiac catheterization. The nurse explains to the client that this procedure is being used in this specific case to: A Open and dilate the blocked coronary arteries B Assess the extent of arterial blockage C Bypass obstructed vessels D Assess the functional adequacy of the valves and heart muscle

B. Cardiac catheterization is done in clients with angina primarily to assess the extent and severity of the coronary artery blockage, A decision about medical management, angioplasty, or coronary artery bypass surgery will be based on the catheterization results.

What are the anti-aginal actions of organic nitrates? A. Decrease O2 consumption; redistribution of coronary flow to infarct areas; relief of coronary spasm; improve perfusion B. Decrease O2 consumption; redistribution of coronary flow to ischemic areas; relief of coronary spasm; improve perfusion C. Increase O2 consumption; redistribution of coronary flow to infarct areas; relief of coronary spasm; improve perfusion to other organs

B. Decrease O2 consumption; redistribution of coronary flow to ischemic areas; relief of coronary spasm; improve perfusion

The nurse prepares a discharge teaching plan for a 44-year-old male patient who has recently been diagnosed with coronary artery disease. Which risk factor should the nurse plan to focus on during the teaching session? A. Type A personality B. Elevated serum lipids C. Family cardiac history D. Hyperhomocysteinemia

B. Elevated serum lipids Dyslipidemia is one of the four major modifiable risk factors for coronary artery disease (CAD). The other major modifiable risk factors are hypertension, tobacco use, and physical inactivity. Research findings related to psychologic states (i.e., type A personality) as a risk factor for coronary artery disease have been inconsistent. Family history is a nonmodifiable risk factor. High homocysteine levels have been linked to an increased risk for CAD.

An infant with tetralogy of Fallot becomes cyanotic and dyspneic after a crying episode. To relieve the cyanosis and dyspnea, the nurse should place the infant in the: A. Orthopneic position B. Knee-chest position C. Lateral Sims' position D. Semi-Fowler's position

B. Flexing the hips and knees decreases venous return to the heart from the legs; when venous return to the heart is decreased, the cardiac workload is decreased.

An occupational health nurse is triaging clients at the site of an industrial explosion. A class III or green tag, should be assigned to client with which of the following manifestations? A. Open femur fracture B. Facial contusions C. Hypovolemic Shock D. Penetrating spinal injury

B. Green "good to go". A yellow tag for open wounds. C and D red tags for emergent care.

The mother of a child with a congenital cardiac defect asks the nurse why her child squats after exertion. The nurse should reply that this position: A. Reduces muscle aches B. Increases cardiac efficiency C. Enhances the pull of gravity D. Decreases blood volume in the extremities

B. When the child squats, blood pools in the lower extremities because of flexion of the hips and knees; less blood returns to the hear, enabling the heart to beat more effectively.

A client who is scheduled for permanent transvenous pacemaker insertion says to the nurse, "I know I need it, but I'm not sue this surgery is the best idea." Which nursing response will best help the nurse assess the client's preoperative concerns? a) how does your family feel about the surgery? b) has anyone taught you about the procedure yet? c) you sound uncertain about the procedure. Can you tell me more about what has you concerned? d) you sound unnecessarily worried. Has anyone told you that the technology is quite advanced now?

C - Anxiety is common in the client with the need for pacemaker insertion. This can be related to a fear of life-threatening dysrhythmias or of the surgical procedure. Option A is not indicated, because it asks about the family and deflects attention away from the client's concerns. Options B and D are close-ended and not exploratory. Option C is open-ended and uses clarification as a communication technique to explore the client's concerns.

A spouse of a client who is scheduled for the insertion of an implantable cardioverter-defibrillator (ICD) expresses anxiety about what would happen if the device discharges during physical contact. The nurse tells the spouse that: a) physical contact should be avoided whenever possible b) the spouse would not feel or be harmed by the countershock c) the shock would be felt, but it would not cause the spouse any harm d) a warning device sounds before countershock, so there is time to move away

C - Clients and families are often fearful about the activation of the ICD. Their fears are about the device itself and also about the occurrence of life-threatening dysrhythmias that trigger its function. Family members need reassurance that, even if the device activates while they are touching the client, the level of the charge is not high enough to harm the family member, although it will be felt. The ICD emits a warning beep when the client is near magnetic fields, which could possibly deactivate it, but it does not beep before countershock

A 57-year-old client with a history of asthma is prescribed propranolol (Inderal) to control hypertension. Before administered propranolol, which of the following actions should the nurse take first? A Monitor the apical pulse rate B Instruct the client to take medication with food C Question the physician about the order D Caution the client to rise slowly when standing

C Propranolol and other beta-adrenergic blockers are contraindicated in a client with asthma, so the nurse should question the physician before giving the dose. The other responses are appropriate actions for a client receiving propranolol, but questioning the physician takes priority. The client's apical pulse should always be checked before giving propranolol; if the pulse rate is extremely low, the nurse should withhold the drug and notify the physician.

A nurse is triaging clients after a mass casualty. Which of the following clients should the nurse assess first? a. a client who has a splinted open fracture of left medial malleolus b. a client who has a penetrating head injury and is experiencing seizures. c. A client who has severe respiratory stridor and a deviated trachea d. a client who has a small circular partial thickness burn to the left calf.

C - this is emergent. The client can survive if immediate care is given. Client A is expectant because his injuries are incompatible with life even if he receives care. Client B is urgent, but can wait a little. Client D is non urgent and can wait hours in a mass casualties.

Medical treatment of coronary artery disease includes which of the following procedures? A Cardiac catheterization B Coronary artery bypass surgery C Oral medication administration D Percutaneous transluminal coronary angioplasty

C. Oral medication administration is a noninvasive, medical treatment for coronary artery disease. Cardiac catheterization isn't a treatment but a diagnostic tool. Coronary artery bypass surgery and percutaneous transluminal coronary angioplasty are invasive, surgical treatments.

A 1-year-old with postductal coarctation of the aorta is admitted to the acute care unit for treatment. When performing an assessment, the nurse finds that the lower extremities are cool. Which finding should the nurse anticipate as the assessment continues? A. Lethargy B. Low blood pressure in the arms C. Low blood pressure in the legs D. Bilateral pedal edema

C. Postductal coarctation of the aorta causes several changes in the lower extremities: diminished peripheral pulses, hypotension, and resulting cool temp. A child under age 3 can't describe his symptoms, but may exhibit exceptional irritability (rather than lethargy). High blood pressure in the upper portions of the body produces headache and vertigo. Pedal edema isn't related to diminished perfusion of the lower extremities.

Prolonged occlusion of the right coronary artery produces an infarction in which of the following areas of the heart? A Anterior B Apical C Inferior D Lateral

C. The right coronary artery supplies the right ventricle, or the inferior portion of the heart. Therefore, prolonged occlusion could produce an infarction in that area. The right coronary artery doesn't supply the anterior portion (left ventricle), lateral portion (some of the left ventricle and the left atrium), or the apical portion (left ventricle) of the heart.

To evaluate a client's condition following cardiac catheterization, the nurse will palpate the pulse: A In all extremities B At the insertion site C. Distal to the catheter insertion D Above the catheter insertion

C. Palpating pulses distal to the insertion site is important to evaluate for thrombophlebitis and vessel occlusion. They should be bilateral and strong.

Sublingual nitroglycerin tablets begin to work within 1 to 2 minutes. How should the nurse instruct the client to use the drug when chest pain occurs? A Take one tablet every 2 to 5 minutes until the pain stops. B Take one tablet and rest for 10 minutes. Call the physician if pain persists after 10 minutes. C Take one tablet, then an additional tablet every 5 minutes for a total of 3 tablets. Call the physician if pain persists after three tablets. D Take one tablet. If pain persists after 5 minutes, take two tablets. If pain still persists 5 minutes later, call the physician.

C. The correct protocol for nitroglycerin used involves immediate administration, with subsequent doses taken at 5-minute intervals as needed, for a total dose of 3 tablets. Sublingual nitroglycerin appears in the bloodstream within 2 to 3 minutes and is metabolized within about 10 minutes.

In order to prevent the development of tolerance, the nurse instructs the patient to: A Apply the nitroglycerin patch every other day B Switch to sublingual nitroglycerin when the patient's systolic blood pressure elevates to >140 mm Hg C Apply the nitroglycerin patch for 14 hours each and remove for 10 hours at night D Use the nitroglycerin patch for acute episodes of angina only

C. Tolerance can be prevented by maintaining an 8- to 12-hour nitrate-free period each day.

Which of the following terms refers to chest pain brought on by physical or emotional stress and relieved by rest or medication? A. Ischemia B. Atherosclerosis C. Angina pectoris D. Atheroma

C. Angina pectoris is a symptom of myocardial ischemia.Atherosclerosis is an abnormal accumulation of lipid deposits and fibrous tissue within arterial walls and lumens. Atheromas are fibrous caps composed of smooth muscle cells that form over lipid deposits within arterial vessels. Ischemia is insufficient tissue oxygenation and may occur in any part of the body.

Postoperative care of a patient undergoing coronary artery bypass graft (CABG) surgery includes monitoring for what common complication? A. Dehydration B. Paralytic ileus C. Atrial dysrhythmias D. Acute respiratory distress syndrome

C. Atrial dysrhythmias Postoperative dysrhythmias, specifically atrial dysrhythmias, are common in the first 3 days following CABG surgery. Although the other complications could occur, they are not common complications.

A 3 month old infant is admitted with a diagnosis of tetralogy of Fallot. Assessment reveals that the infant's weight is in the 5th percentile. The nurse is aware that the reason for this inadequate weight gain is: A. Cyanosis leading to cerebral changes B. Decreased arterial Po2 resulting in polycythemia C. Activity intolerance resulting in deficient caloric intake D. Pulmonary hypertension resulting in recurrent respiratory infections.

C. Because the infant tires so easily, sufficient calories cannot be infested to meet nutritional needs.

An appropriate nursing action to include in the care of an infant with congenital heart disease who has been admitted with heart failure is: A. Positioning flat on the back B. Encouraging nutritional fluids C. Offering small frequent feedings D. Measuring the head circumference

C. Because these infants become extremely fatigued while sucking, small frequent feedings with adequate rest periods can improve their total intake.

The nurse assesses the right femoral artery puncture site as soon as the patient arrives after having a stent inserted into a coronary artery. The insertion site is not bleeding or discolored. What should the nurse do next to ensure the femoral artery is intact? A. Palpate the insertion site for induration. B. Assess peripheral pulses in the right leg. C. Inspect the patient's right side and back. D. Compare the color of the left and right legs.

C. Inspect the patient's right side and back. The best method to determine that the right femoral artery is intact after inspection of the insertion site is to logroll the patient to inspect the right side and back for retroperitoneal bleeding. The artery can be leaking and blood is drawn into the tissues by gravity. The peripheral pulses, color, and sensation of the right leg will be assessed per agency protocol.

During the previous few months, a 56-year-old woman felt brief twinges of chest pain while working in her garden and has had frequent episodes of indigestion. She comes to the hospital after experiencing severe anterior chest pain while raking leaves. Her evaluation confirms a diagnosis of stable angina pectoris. After stabilization and treatment, the client is discharged from the hospital. At her follow-up appointment, she is discouraged because she is experiencing pain with increasing frequency. She states that she is visiting an invalid friend twice a week and now cannot walk up the second flight of steps to the friend's apartment without pain. Which of the following measures that the nurse could suggest would most likely help the client deal with this problem? A Visit her friend earlier in the day B Rest for at least an hour before climbing the stairs C Take a nitroglycerin tablet before climbing the stairs D Lie down once she reaches the friend's apartment

C. Nitroglycerin may be used prophylactically before stressful physical activities such as stair climbing to help the client remain pain free. Visiting her friend early in the day would have no impact on decreasing pain episodes. Resting before or after an activity is not as likely to help prevent an activity-related pain episode.

As an initial step in treating a client with angina, the physician prescribes nitroglycerin tablets, 0.3mg given sublingually. This drug's principal effects are produced by: A Antispasmodic effect on the pericardium B Causing an increased myocardial oxygen demand C Vasodilation of peripheral vasculature D Improved conductivity in the myocardium

C. Nitroglycerin produces peripheral vasodilation, which reduces myocardial oxygen consumption and demand. Vasodilation in coronary arteries and collateral vessels may also increase blood flow to the ischemic areas of the heart. Nitroglycerin decreases myocardial oxygen demand. Nitroglycerin does not have an effect on pericardial spasticity or conductivity in the myocardium.

A nurse is preparing to administer a new prescription for isoniazid (INH) to a client who has TB. Which of the following is an appropriate statement by the nurse about this medication? a. you may notice yellowing of your skin b. you may experience pain in your joints c. you may notice tingling of your hands d. you may experience a loss of appetite

C. Peripheral neuritis is adverse effect of INH. Take vitamin B6 (pyroxidine). Yellowing of skin common too, but with all anti tubercular drugs. B6/peripheral neuritis more specific to INH

Mariah has been diagnosed with acquired valvular heart disease. The primary cause of acquired valvular heart disease is a. heredity b. drug abuse c. rheumatic fever d. fetal alcohol syndrome

C. Rheumatic fever remains the most common cause of acquired valvular heart disease. The valves are a perfect place for bacteria to colonize. Blood is a perfect medium for bacterial growth. The causative organism is beta-hemolytic streptococcus.

The nurse is teaching a patient who is starting anti tubercular therapy with rifampin. Which adverse effects would the nurse expect to see? a. head ache and neck pain b. gynecomastia c. reddish brown urine d. numbness or tingling of extremities

C. Rifampin (RIF) R=Red (urine, tears, saliva, sweat, secretions), and not REDy to have a baby (counteracts birth control). **Reddish/orangy urine normal. If gets darker (jaundice-like brown, can indicate liver dysfunction - Rifampin also affects liver and can cause peripheral neuritis.

A 4-month old infant who has a congenital heart defect develops heart failure and is exhibiting marked dyspnea at rest . The nurse is aware this finding can be attributed to: A. Anemia B. Hypovolemia C. Pulmonary edema D. Metabolic acidosis

C. The increased blood volume and pressure in the lungs resulting from left ventricular failure causes pulmonary edema; dyspnea, and early sign of failure, is probably caused by the decreased distensibility of the lungs.

What are the effects of organic nitrates? A. Vasodilation; dilation of large veins results in decreased pre-load and decreased blood pressure B. Vasodilation; dilation of large veins results in decreased after-load and decreased cardiac output C. Vasodilation; dilation of large veins results in decreased pre-load and decreased cardiac output

C. Vasodilation; dilation of large veins results in decreased pre-load and decreased cardiac output

When teaching a client with a cardiac problem, who is on a high-unsaturated fatty-acid diet, the nurse should stress the importance of increasing the intake of: A Enriched whole milk B Red meats, such as beef C. Vegetables and whole grains D Liver and other glandular organ meats

C. Vegetables and whole grains are low in fat and may reduce the risk for heart disease.

A nurse caring for a client who has a new diagnosis of TB and has been placed on a multi medication regimen. Which of the following instructions should the nurse give the client related to the medication ethambutol? a. your urine may be a dark orange color b. watch for change in the sclera of your eyes c. watch for any vision changes d. take vitamin B6 daily

C. Watch for vision changes due to optic neuritis which can result from taking this medication

A nurse is teaching the mother of an infant who will take digoxin (Lanoxin) at home to treat a chronic tachyarrhythmia. Which signs of digoxin toxicity should the mother be taught? A. Blurred vision B. Heart rate of 180 beats/minute C. Vomiting two or more feedings D. Bulging of the anterior fontanel

C. signs of digoxin toxicity include nausea, vomiting, blurred vision, and yellow-green visual spots, but the mother will only be able to assess objective symptoms such as vomiting. Digoxin causes a decreased heart rate, which can progress to complete heart block if toxicity occurs (digoxin toxicity doesn't lead to tachycardia). Bulging of the anterior fontanel is a sign of increased intracranial pressure.

Which of the following laboratory findings support the diagnosis of disseminated intravascular coagulopathy (DIC)? a) elevated factor assays (II, V, and VII) b) increased platelet count c) elevated RBC, WBC, platelets d) prolonged prothrombin time and partial thromboplastin time

D - DIC - is body's response to overstimulation of clotting and articulating processes in response to injury or disease. In DIC, bleeding occurs due to depletion of platelets in the general circulation which is due to massive blood clotting (decreased fibrinogen, increased protime, increased PTT, decreased platelets).

What is the primary reason for administering morphine to a client with myocardial infarction? A To sedate the client B To decrease the client's pain C To decrease the client's anxiety D To decrease oxygen demand on the client's heart

D. Morphine is administered because it decreases myocardial oxygen demand. Morphine will also decrease pain and anxiety while causing sedation, but isn't primarily given for those reasons.

To enhance the percutaneous absorption of nitroglycerine ointment, it would be MOST important for the nurse to select a site that is A.Muscular. B.Near the heart. C.Over a bony prominence. D.Non-hairy

D. Nitroglycerine: used in treatment of angina pectoris to reduce ischemia and relieve pain by decreasing myocardial oxygen consumption; dilates veins and arteries. Side effects: throbbing headache, flushing, hypotension, tachycardia. Nursing responsibilities: teach appropriate administration, storage, expected pain relief, side effects. Ointment applied to skin; sites rotated to avoid skin irritaion. Prolonged effect up to 24 hours.

When preparing a client for discharge after surgery for a CABG, the nurse should teach the client that there will be: A No further drainage from the incisions after hospitalizations B A mild fever and extreme fatigue for several weeks after surgery C Little incisional pain and tenderness after 3 to 4 weeks after surgery D Some increase in edema in the leg used for the donor graft when activity increases

D. The client is up more at home, so dependent edema usually increases. Serosanguineous drainage may persist after discharge.

The nurse instructs a 68-year-old woman with hypercholesterolemia about natural lipid-lowering therapies. The nurse determines further teaching is necessary if the patient makes which statement? A. "Omega-3 fatty acids are helpful in reducing triglyceride levels." B. "I should check with my physician before I start taking any herbal products." C. "Herbal products do not go through as extensive testing as prescription drugs do." D. "I will take garlic instead of my prescription medication to reduce my cholesterol."

D. "I will take garlic instead of my prescription medication to reduce my cholesterol." Current evidence does not support using garlic in the treatment of elevated cholesterol. Strong evidence supports the use of omega-3 fatty acids for reduction of triglyceride levels. Many herbal products are not standardized and effects are not predictable. Patients should consult with their health care provider before starting herbal or natural therapies.

A paradoxical pulse occurs in a client who had a coronary artery bypass graft (CABG) surgery 2 days ago. Which of the following surgical complications should the nurse suspect? A Left-sided heart failure B Aortic regurgitation C Complete heart block D Pericardial tamponade

D. A paradoxical pulse (a palpable decrease in pulse amplitude on quiet inspiration) signals pericardial tamponade, a complication of CABG surgery. Left-sided heart failure can cause pulsus alternans (pulse amplitude alternation from beat to beat, with a regular rhythm). Aortic regurgitation may cause bisferious pulse (an increased arterial pulse with a double systolic peak). Complete heart block may cause a bounding pulse (a strong pulse with increased pulse pressure).

The patient is being dismissed from the hospital after ACS and will be attending rehabilitation. What information does the patient need to be taught about the early recovery phase of rehabilitation? A. Therapeutic lifestyle changes should become lifelong habits. B. Physical activity is always started in the hospital and continued at home. C. Attention will focus on management of chest pain, anxiety, dysrhythmias, and other complications. D. Activity level is gradually increased under cardiac rehabilitation team supervision and with ECG monitoring.

D. Activity level is gradually increased under cardiac rehabilitation team supervision and with ECG monitoring. In the early recovery phase after the patient is dismissed from the hospital, the activity level is gradually increased under supervision and with ECG monitoring. The late recovery phase includes therapeutic lifestyle changes that become lifelong habits. In the first phase of recovery, activity is dependent on the severity of the angina or MI, and attention is focused on the management of chest pain, anxiety, dysrhythmias, and other complications. With early recovery phase, the cardiac rehabilitation team may suggest that physical activity be initiated at home, but this is not always done.

For which problem is percutaneous coronary intervention (PCI) most clearly indicated? A. Chronic stable angina B. Left-sided heart failure C. Coronary artery disease D. Acute myocardial infarction

D. Acute myocardial infarction PCI is indicated to restore coronary perfusion in cases of myocardial infarction. Chronic stable angina and CAD are normally treated with more conservative measures initially. PCI is not relevant to the pathophysiology of heart failure, such as left-sided heart failure.

The nurse is caring for a child who has undergone a cardiac catheterization. During recovery, the nurse notices the dressing is saturated with bright red blood. The nurse's first action is to: A. Call the interventional cardiologist. B. Notify the cardiac catheterization laboratory that the child will be returning C. Apply a bulky pressure dressing over the present dressing D. Apply direct pressure 1 inch above the puncture site will localize pressure over the vessel site.

D. Apply direct pressure 1 inch above the puncture site will localize pressure over the vessel site.

A patient was admitted to the emergency department (ED) 24 hours earlier with complaints of chest pain that were subsequently attributed to ST-segment-elevation myocardial infarction (STEMI). What complication of MI should the nurse anticipate? A. Unstable angina B. Cardiac tamponade C. Sudden cardiac death D. Cardiac dysrhythmias

D. Cardiac dysrhythmias The most common complication after MI is dysrhythmias, which are present in 80% of patients. Unstable angina is considered a precursor to MI rather than a complication. Cardiac tamponade is a rare event, and sudden cardiac death is defined as an unexpected death from cardiac causes. Cardiac dysfunction in the period following an MI would not be characterized as sudden cardiac death.

While monitoring the patient, the nurse knows that a therapeutic response to an anti tubercular antibiotic would be: a. the patient states that he/she feeling much better b. patient lab results show lower WBC c. patient reports decrease in cough and night sweats d. decrease in symptoms, along with improved chest radiography and sputum culture results.

D. Sputum test - AFB (acid fast bacillus) - indicates active TB. The definitive test. Mantoux (skin test) just indicates that client has been exposed, but can be dormant for years. Only contagious when active.

When counseling a patient who has been newly diagnosed with TB, the nurse will make sure that the patient realizes that he or she is contagious a. during all phases of the illness b. any time up to 18 months after therapy begins c. during the postictal phase of TB d. during the initial period of the illness and its diagnosis.

D. The client is no longer contagious after 3 negative sputum samples (AFB) (amount 1-2 months after antibiotics started)

A one month old infant is admitted for confirmation of the diagnosis of ventricular septal defect. During the initial admission assessment, the nurse would expect to find: A. Bradycardia at rest B. Bounding peripheral pulses C. An activity related cyanosis D. A murmur at the left sternal border

D. This murmur is the most characteristic finding in children with VSD

What congenital heart defect causes cyanosis in children? A. Atrial septal defect B. Coarctation of the aorta C. Ventricular septal defect D. Trasposition of the great vessels

D. With transposition of the great vessels, the pulmonary artery is attached to the left ventricle and the aorta is attached to the right ventricle. The child is cyanotic because blood reaches the tissues from the right ventricle before being oxygenated by the lungs. In atrial septal defect and ventricular septal defect, blood is shunted from the left side of the heart to the right side through patent openings. Because the blood travels from left to right, it's oxygenated and doesn't produce cyanosis. Coarctation of the aorta is a narrowing of the aorta that decreases the circulation of oxygenated blood to the body. With this condition, the child won't be cyanotic unless cardiac output drops.

Luke is a 65 year old admitted to ER for treatment of chest pain and severe hypertension. A nitroglycerin drip was ordered. Which of the following is true regarding nitroglycerin administration? a. use only plastic IV tubing b. NTG may be piggybacked to Isolate P c. NTG should be filtered prior to delivery d. have normal saline or other volume expander available at bedside

D. when administering NTG, it is important to have NS or other volume expander at bedside incase of vascular collapse. The potential collapse is related to peripheral venous and arterial dilation (relative hypovolemia). Only non polyvinyl chloride tubing may be used for infusion because PVC may be absorbed by the NTG. Filtration is not required before infusion. NTG is only compatible with D5W, NS, LR, D5NS, and half normal saline.

The 4 "D's" of Eppiglottitis are: Dysphonia (muffled voice), Drooling, distressed respiratory status and ___________ (impairment of speech)

Dysphasia

A 3-month-old has been diagnosed with a ventricular septal defect (VSD). The flow of blood through the heart is _____________________.

Left to right. The pressures in the left side of the heart are greater, causing the flow of blood to be from an area of higher pressure to lower pressure, or left to right, increasing the pulmonary blood flow with the extra blood

Goal for child with respiratory infection: "The child will exhibit adequate oxygenation and __________ airway. Keep O2 sats >92-93%

Patent

Indomethacin may be given to close which congenital heart defect (CHD) in newborns? _____________________

Patent ductus arteriosus or PDA

A newborn is diagnosed with a congenital heart defect (CHD). The test results reveal that the lumen of the duct between the aorta and pulmonary artery remains open. This defect is known as _____________________.

Patent ductus arteriosus or PDA.

Which of the following diagnostic tools is most commonly used to determine the location of myocardial damage? A Cardiac catheterization B Cardiac enzymes C Echocardiogram D. Electrocardiogram (ECG)

The ECG is the quickest, most accurate, and most widely used tool to determine the location of myocardial infarction. Cardiac enzymes are used to diagnose MI but can't determine the location. An echocardiogram is used most widely to view myocardial wall function after an MI has been diagnosed. Cardiac catheterization is an invasive study for determining coronary artery disease and may also indicate the location of myocardial damage, but the study may not be performed immediately.

IV heparin therapy is ordered for a client. While implementing this order, a nurse ensures that which of the following medications is available on the nursing unit? A Vitamin K B Aminocaproic acid C Potassium chloride D Protamine sulfate

The antidote to heparin is protamine sulfate and should be readily available for use if excessive bleeding or hemorrhage should occur. Vitamin K is an antidote for warfarin.

How is TB treated? A. Antiviral drugs B. Antibiotics C. Surgery D. Chemotherapy

The correct answer is B. Antibiotics. Treatment usually combines several antibiotics that you take for 6 to 12 months. It's important that you take your medicine on time every day. WIf you stop taking the medicine before the end of treatment, you risk having TB return. TB that returns may be resistant to drugs used to treat it. This makes it much more difficult to cure. If you have a positive skin test for TB, but no other signs of the disease, your doctor may ask you to take an antibiotic for 9 months to kill any TB bacteria in your body and prevent development of active TB in the future.

When planning the response to the potential use of smallpox as the agent of terrorism, the emergency department nurse manager will plan to obtain sufficient quantities of ______________________.

Vaccine. - no cure for smallpox, but vaccine can prevent it even sometimes if given rapidly after exposure

A nurse is precasting a newly licensed nurse caring for a patient suspected of having TB. The nurse instructs the newly nurse that the standard infection control procedures in this situation include A. Placing client on droplet precautions B. Placing the client in a room with positive pressure airflow C. Wearing surgical masks D. Wearing particulate respirator masks

D - nurse should wear highly efficient respirator masks (N95)

Nurse preparing for admision of infant w/diagnosis of bronchiolitis cause by RSV. Choose intervention that would be included in plan of care. (select all) 1. Place the infant in a private room. 2. Ensure that the infant's head is in a flexed position. 3. Wear a mask at all times when in contact with the infant. 4. Place the infant in a tent that delivers warm humidified air. 5. Position the infant on the side, with the head lower than the chest. 6. Ensure that nurses caring for the infant with RSV do not care for other high-risk children.

1 & 6 Place infant in private room. Place infant in room near nurses' station. R: Infant w/RSV should be isolated in a private room or in a room w/another child w/RSV. Infant s/be placed in room near nurses' station for easy observation. Infant s/be positioned w/head and chest at 30 to 40 degree angle and neck slightly extended to maintain an open airway and to decrease pressure on diaphragm. Cool, humidified oxygen is delivered to relieve dyspnea, hypoxemia, and insensible water loss from tachypnea. Contact precautions (wearing gloves and a gown) reduce nosocomial transmission of RSV.

The nurse reinforces instructions to mother of child w/croup about measures to take if an acute spasmodic episode occcurs. Which statment by mom indicates need for further instruction? 1. "I should place a steam vaporizer in my child's room." 2. "I will take my child out into the cool, humid night air." 3. "I could place a cool mist humidifier in my child's room." 4. "I will have my child inhale the steam from warm running water."

1. I will place a steam vaporizer in my child's room. R: Steam from warm running water in a closed bathroom and cool mist from a bedside humidifier are effective for reducing mucosal edema. Cool-mist humidifiers are recommended as compared w/steam vaporizers, which present a danger of scalding burns. Taking the child out into the cool, humid night air may also relieve mucosal swelling.

Nurse working in ER caring for child who has been diagnosed w/epiglottitis. Indications that child may be experiencing airway obstruction include which of following? 1. The child exhibits nasal flaring and bradycardia. 2. The child is leaning forward, with the chin thrust out. 3. The child has a low-grade fever and complains of a sore throat. 4. The child is leaning backward, supporting himself or herself with the hands and arms.

3. "tripod" The child thrusts the chin forward and opens the mouth. R: Clinical manifestations that are suggestive of airway obstruction include tripod positioning (leaning forward supported by the hands and arms w/chin trhust out and mouth open), nasal flaring, tachcardia, a high fever, and a sore throat.

A client diagnosed with active TB would be hospitalized primarily for which of the following reasons? A To evaluate his condition B To determine his compliance C To prevent spread of the disease D To determine the need for antibiotic therapy

Question 19 Explanation: C The client with active TB is highly contagious until three consecutive sputum cultures are negative, so he's put in respiratory isolation in the hospital.

A 2 year old is admitted with croup and 1/4L oxygen via nasal canula is administered because it: A. Congeals the mucous secretions and relieves the dyspnea B. Decreases the effort required for breathing and also allows for rest. C. Triggers the cough reflex and facilitates expectoration of mucus D. Liquifies the mucous secretions and makes them easier to expectorate

B. Administering oxygen via nasal cannula limits the energy required for breathing, thus allowing the child to conserve energy that can be used for fluid and nutrient intake

Which of the following pathophysiological mechanisms that occurs in the lung parenchyma allows pneumonia to develop? A Atelectasis B Bronchiectasis C Effusion D Inflammation

Question 2 Explanation: D The common feature of all type of pneumonia is an inflammatory pulmonary response to the offending organism or agent. Atelectasis and bronchiectasis indicate a collapse of a portion of the airway that doesn't occur in pneumonia. An effusion is an accumulation of excess pleural fluid in the pleural space, which may be a secondary response to pneumonia.

An 8-year-old is admitted with drooling, muffled phonation, and a temperature of 102°F. The nurse should immediately notify the doctor because the child's symptoms are suggestive of: A Strep throat B Epiglottitis C Laryngotracheobronchitis D Bronchiolitis

B. The child's symptoms are consistent with those of epiglottitis, an infection of the upper airway that can result in total airway obstruction. Symptoms of strep throat, laryngotracheobronchitis, and bronchiolitis are different than those presented by the client; therefore, answers A, C, and D are incorrect.

How do you get TB? A. Through the air B. Through sexual contact C. Through contaminated food D. Through blood E. A and C

A - through the air **(Airborne and droplet - gown, gloves, N95 mask) Tuberculosis is a disease that can affect any part of the body, but the infection most often targets the lungs. Most TB cases are caused by inhaling the bacterium. People can get TB when an infected person coughs, sneezes, speaks, sings, or laughs. Although tuberculosis has been around for thousands of years, it did not become a major health problem until the Industrial Revolution, when crowded living conditions helped it spread. In the 17th and 18th centuries, TB caused a quarter of all adult deaths in Europe. Only the active form of the disease is contagious. Some people have inactive TB, or latent TB. This can't be spread to other people unless it becomes activated for some reason.

Which of the following mental status changes may occur when a client with pneumonia is first experiencing hypoxia? A Coma B Apathy C Irritability D Depression

Question 47 Explanation: C Clients who are experiencing hypoxia characteristically exhibit irritability, restlessness, or anxiety as initial mental status changes. As the hypoxia becomes more pronounced, the client may become confused and combative. Coma is a late clinical manifestation of hypoxia. Apathy and depression are not symptoms of hypoxia.

A nurse is developing an educational program on bioterrorism and smallpox. The nurse should include in the teaching to expect the smallpox rash to first appear in which of the following areas? A. Face B. Posterior shoulders C. Abdomen D. Lower extremities

A. No cure for smallpox. Vaccine!!! Prevent.

A 2 year old is suffering from poor oxygenation due to streptococcal pneumonia and requires an endotracheal (ET) tube. Which outcome indicates that ET intubation is successful? A. Bilateral breath sounds on auscultation are heard B. Oxygen saturation is 80% C. An audible cry is heard on inspiration D. The tip of the ET tube is in the right mainstem bronchus.

A. Bilateral breath sounds heard

A nurse notes 2+ bilateral edema in the lower extremities of a client with myocardial infarction who was admitted 2 days ago. The nurse would plan to do which of the following next? A Review the intake and output records for the last 2 days B Change the time of diuretic administration from morning to evening C Request a sodium restriction of 1 g/day from the physician D Order daily weights starting the following morning

A. Edema, the accumulation of excess fluid in the interstitial spaces, can be measured by intake greater than output and by a sudden increase in weight. Diuretics should be given in the morning whenever possible to avoid nocturia. Strict sodium restrictions are reserved for clients with severe symptoms.

In the work setting, what is your primary responsibility in preparing for disaster management that includes natural disasters or bioterrorism incidents? A Knowledge of the agency's emergency response plan B Awareness of the signs and symptoms for potential agents of bioterrorism C Knowledge of how and what to report to the CDC D Ethical decision-making about exposing self to potentially lethal substances

A. In preparing for disasters, the RN should be aware of the emergency response plan. The plan gives guidance that includes roles of team members, responsibilities, and mechanisms of reporting. Signs and symptoms of many agents will mimic common complaints, such as flu-like symptoms. Discussions with colleagues and supervisors may help the individual nurse to sort through ethical dilemmas related to potential danger to self.

A nurse is planning a community education program on the transmission of tuberculosis. According to the epidemiological triangle, which of the following is mycobacterium tuberculosis classified as? A. Agent B. Host C. Environment D. Resevior

A. The agent is the biological, chemical or physical entity that is responsible for the health alteration.

There has been a massive community disaster and stable patients must be discharged from the hospital to make room for the influx of new casualties. Which of the following clients is safe to discharge? A. A client with multiple sclerosis who reports ataxia B. A client with deep vein thrombosis whose Ptt is normal C. A client with right lower quadrant pain and positive rebound tenderness D. Client whose amylase and lipase are twice the normal value

A. chronic MS and ataxia is normal finding

A 60-year-old male client comes into the emergency department with complaints of crushing chest pain that radiates to his shoulder and left arm. The admitting diagnosis is acute myocardial infarction. Immediate admission orders include oxygen by NC at 4L/minute, blood work, chest x-ray, an ECG, and 2 mg of morphine given intravenously. The nurse should first: A Administer the morphine B Obtain a 12-lead ECG C Obtain the lab work D Order the chest x-ray

A.Although obtaining the ECG, chest x-ray, and blood work are all important, the nurse's priority action would be to relieve the crushing chest pain.

A client is wearing a continuous cardiac monitor, which begins to sound its alarm. A nurse sees no electrocardiogram complexes on the screen. The first action of the nurse is to: A Check the client status and lead placement B Press the recorder button on the electrocardiogram console C Call the physician D Call a code blue

A.Sudden loss of electrocardiogram complexes indicates ventricular asystole or possible electrode displacement. Accurate assessment of the client and equipment is necessary to determine the cause and identify the appropriate intervention.

The nurse is teaching a client who has been diagnosed with TB how to avoid spreading the disease to family members. Which statement(s) by the client indicate(s) that he has understood the nurses instructions? Select all that apply. A "I will need to dispose of my old clothing when I return home." B "I should always cover my mouth and nose when sneezing." C "It is important that I isolate myself from family when possible." D "I should use paper tissues to cough in and dispose of them properly." E "I can use regular plate and utensils whenever I eat."

B, D, and website says E but check notes - thought needed to be disposable

A 5 year old child is brought into the emergency department with drooling, strident cough, and lethargy. Epiglottitis is suspected. The priority intervention for this child is to: A. Take vital signs B. Secure the child's airway C. Visualize the child's throat with a tongue depressor D. Obtain throat cultures

B. The priority in treating epiglottitis is achieving a patent airway as quickly as possible.

A 15-month old with croup is admitted to the pediatric unit. The nurse is most concerned that: 1. An inspiratory stridor is heard 2. The mother cannot calm the child 3. The toddler has a barking cough 4. The toddler is restless while sleeping

B. When a mother cant calm a child with a respiratory problem, assess for increasing hypoxia. Normal symptoms of croup include inspiratory stridor and barking cough. Children are commonly restless in their sleep when ill.

A client has driven himself to the ER. He is 50 years old, has a history of hypertension, and informs the nurse that his father died of a heart attack at 60 years of age. The client is presently complaining of indigestion. The nurse connects him to an ECG monitor and begins administering oxygen at 2 L/minute per NC. The nurse's next action would be to: A Call for the doctor B Start an intravenous line C Obtain a portable chest radiograph D Draw blood for laboratory studies

B. Advanced cardiac life support recommends that at least one or two intravenous lines be inserted in one or both of the antecubital spaces. Calling the physician, obtaining a portable chest radiograph, and drawing blood are important but secondary to starting the intravenous line.

A community health nurse suspects an outbreak of scabies. What action should the nurse take first? A. Educate the community on the disease transmission B. Determine the incidence rate C. Institute prophylactic treatment D. Discuss treatment plans with client families

B. Use nursing process to determine rate of new cases

A nurse is working in the ER when there is an explosion at a local chemical plant. Multiple victims are expected at the facility. Which of the following should be the nurses priority intervention upon client arrival? A. Call for additional help to support increased client volume B. Isolate the patients and irrigate with water C. Provide an area for family members to wait D. Obtain medical histories of each client

B. Use urgent vs. non-urgent to reduce morbidity and mortality

A 4-year old female client is brought to the emergency room after waking up with bark-like cough and stridor. On arrival to the ER, she has respiratory distress and is afebrile. The diagnosis is croup. What instruction should you give the parents? a) perform percussion and postural drainage before putting the child to bed and before meals b) run a cool mist vaporizer in patient's room during the day c) encourage the child to do coughing and deep breathing exercises d) bring the child to the bathroom and have the tap run with warm water during acute episodes of cough

B. cool mist vaporizer will relieve spasm of airways. This will promote easy breathing of the child with croup

An important nursing measure for a 6-month old infant with bronchiolitis is: Discuss A. Promoting stimulating activities that meet the infant's developmental needs B. Making regular assessments of the infant's skin color, anterior fontanel, and vital signs C. Discouraging visits from the parents during the acute phase to conserve the infant's energy D. Maintaining airborne precautions including wearing a gown, cap, mask, and gloves when giving care to the infant.

B. these assessments are vital to determine the infant's hydration status. Suctioning as needed as well. Stringy white mucus telltale sign. Bronchiolitis = RSV

A nurse is assessing the vital signs of a 3-year old child hospitalized with a diagnosis of croup and notes that the respiratory rate is 28 breaths per minute. Based on this finding, which nursing action is appropriate? a) administer oxygen b) notify the physician c) document the findings d) reassess the respiratory rate in 15 minutes

C - The normal respiratory rate for a 3-year-old child is approximately 20 to 30 breaths per minute. Because the respiratory rate is normal, options A, B, and D are unnecessary actions. The nurse would document the findings.

Which of the following teachings should be given to the mother when her child is in a mist tent for liquefication of secretions? a) give the child a stuff toy inside the mist tent b) avoid nylon blanket inside the mist tent c) advise mother to let the child stay in the mist tent d) give the child coloring book inside the mist tent

C - a child should stay inside the mist tent to liquefy mucous secretions and facilitate breathing. Toys that absorb moisture like stuff toys and coloring book should be avoided. Dampness lowers resistance to infection.

A 4 year old is brought to the emergency department by his parents, who report that he swallowed a small toy. What symptom suggests complete airway obstruction by a foreign body? Discuss A. Gagging B. Coughing C. Inability to speak D. Rapid respirations

C. With complete airway obstruction, the child can't cough, speak, or breathe. Gagging and coughing may be associated with such problems as laryngotracheal obstruction. Bronchial obstruction produces cough, wheezing, syspnea, decreased airway entry, and asymmetrical breath sounds.

A nurse is teaching a client newly diagnosed with active pulmonary tuberculosis about disease transmission. Which of the following should the nurse include in the teaching? A. Household members should be placed in respiratory isolation B. The client should use disposable utensils when eating C. Household members should take Isoniazid (INH) for 6-12 months. D. The client should have a repeat purified protein derivative (PPD) test in 3 months

C. Household members should take INH prophylactically to prevent the transmission to already at risk patients.

An infant with a history of respiratory tract infection is brought to the emergency department and diagnosed with bronchiolitis and respiratory syncytial virus (RSV) infection. The nurse places the infant in a private room and institutes what type of precautions? 1. Droplet precautions 2. Standard precautions 3. Contact precautions 4. Airborne precautions

C. RSV can live on paper for 1 hour and on cribs and other nonporous surfaces for up to 6 hours and is highly communicable. The infant with RSV should be isolated in a single room. Along with standard precautions, the infant should be placed on contact precautions, and gloves and a gown should be worn when entering the room if clothing will contact the infant., Droplet precautions (wearing a mask when entering the room) and airborne precautions (private room with negative air pressure, high efficiency filtration, and shuyt door; visitors required to wear a HEPA-filter mask) aren't necessary.

A client is at risk for pulmonary embolism and is on anticoagulant therapy with warfarin (Coumadin). The client's prothrombin time is 20 seconds, with a control of 11 seconds. The nurse assesses that this result is: A The same as the client's own baseline level B Lower than the needed therapeutic level C Within the therapeutic range D Higher than the therapeutic range

C. The therapeutic range for prothrombin time is 1.5 to 2 times the control for clients at risk for thrombus. Based on the client's control value, the therapeutic range for this individual would be 16.5 to 22 seconds. Therefore the result is within therapeutic range. **WARFARIN should not be mixed with NSAIDS - risk of GI bleed

A client with no history of cardiovascular disease comes into the ambulatory clinic with flu-like symptoms. The client suddenly complains of chest pain. Which of the following questions would best help a nurse to discriminate pain caused by a non-cardiac problem? A "Have you ever had this pain before?" B "Can you describe the pain to me?" C. "Does the pain get worse when you breathe in?" D "Can you rate the pain on a scale of 1-10, with 10 being the worst?"

C.Chest pain is assessed by using the standard pain assessment parameters. Options 1, 2, and 4 may or may not help discriminate the origin of pain. Pain of pleuropulmonary origin usually worsens on inspiration.

The nurse organizes care for an infant with bronchiolitis (RSV) to allow for uniterrupted periods of rest. This plan would be: A. Inappropriate because constant care is necessary in the acute stage B. Appropriate because the cool mist helps to maintain hydration status C. Inappropriate because frequent assessment by auscultation is required D. Appropriate because this action promotes decreased oxygen demands

D. The infant is having difficulty with breathing: disturbing the infant frequently causes an increased expenditure of energy, thus increasing oxygen needs.Cool mist does not promote hydration; it limits inflammation and lowers fever.

A nurse in the ER is admitting clients following an earthquake. The emergency disaster plan has been implemented due to the anticipated high number of casualties. Which of the following clients should the nurse recommend to be seen by the provider immediately. A. A client who has a massive head injury and respirations of 8/min B. A client who has compound open fractures on each leg C. A client who has a 6 in laceration to the scalp. D. A client who has a crushing chest wound.

D. Crushing chest wound is emergent injury that poses immediate threat to life, but has a chance of survival with prompt intervention. B and C can wait, A is not expected to live.

A 10 year old student comes to the school nurse's office. He says he is unable to breathe. The first action is to: A. Take vital signs B. Call the student's mother C. Give an aerosol treatment D. Listen to the student's lungs

D. The child is probably having an asthma attack, and the nurse needs to auscultate his lungs to determine how well he's exchanging air. She should assess for dyspnea, coughing, and wheezing. Vital signs are taken after the pulmonary assessment. The nurse should call the student's mother, but this isn't the first priority. The aerosol treatment is appropriate if prescribed by the child's physician, but it should be done after the nurse assesses the child.

A 9-year-old is admitted with suspected rheumatic fever. Which finding is suggestive of poly migratory arthritis? A Irregular movements of the extremities and facial grimacing B Painless swelling over the extensor surfaces of the joints C Faint areas of red demarcation over the back and abdomen D Swelling, inflammation, and effusion of the joints

D. The child with polymigratory arthritis will exhibit swollen, painful joints. Answer B is incorrect because it describes subcutaneous nodules. Answer C is incorrect because it describes erythema marginatum. Answer A is incorrect because it describes Syndeham's chorea.

A child with croup is placed in a cool, high-humidity tent connected to room air. The primary purpose of the tent is to: A Prevent insensible water loss B Provide a moist environment with oxygen at 30% C Prevent dehydration and reduce fever D Liquefy secretions and relieve laryngeal spasm

D. The primary reason for placing a child with croup under a mist tent is to liquefy secretions and relieve laryngeal spasms. Answer A is incorrect because it does not prevent insensible water loss. Answer B is incorrect because the oxygen concentration is too high. Answer C is incorrect because the mist tent does not prevent dehydration or reduce fever.

A diagnosis of pneumonia is typically achieved by which of the following diagnostic tests? A ABG analysis B Chest x-ray C Blood cultures D Sputum culture and sensitivity

Question 6 Explanation: D Sputum C & S is the best way to identify the organism causing the pneumonia. Chest x-ray will show the area of lung consolidation. ABG analysis will determine the extent of hypoxia present due to the pneumonia, and blood cultures will help determine if the infection is systemic.

Clients with chronic illnesses are more likely to get pneumonia when which of the following situations is present? A Dehydration B Group living C Malnutrition D Severe periodontal disease

Question 1 Explanation: B Clients with chronic illnesses generally have poor immune systems. Often, residing in group living situations increases the chance of disease transmission.

Nurse reviews results of a Mantoux test performed on a 3 yr.old. Reults indicate an area of induration that measures 10 mm. The nurse w/interpret these results as: 1. Positive 2. Negative 3. Inconclusive 4. Definitive and requiring a repeat test

Positive R: An induration that measures 10mm or more is considerd to be a positive result for children who are younger than 4 and for those w/chronic illness or w/a high risk for environmental exposure to TB. A reaction of 5mm or more is considered to be a positive result for those in the highest-risk groups.

A client with primary TB infection can expect to develop which of the following conditions? A Active TB within 2 weeks B Active TB within 1 month C A fever that requires hospitalization D A positive skin test

Question 10 Explanation: D A primary TB infection occurs when the bacillus has successfully invaded the entire body after entering through the lungs. At this point, the bacilli are walled off and skin tests read positive. However, all but infants and immunosuppressed people will remain asymptomatic. The general population has a 10% risk of developing active TB over their lifetime, in many cases because of a break in the body's immune defenses. The active stage shows the classic symptoms of TB: fever, hemoptysis, and night sweats.

A client was infected with TB 10 years ago but never developed the disease. He's now being treated for cancer. The client begins to develop signs of TB. This is known as which of the following types of infection? A Active infection B Primary infection C Superinfection D Tertiary infection

Question 11 Explanation: A Some people carry dormant TB infections that may develop into active disease. In addition, primary sites of infection containing TB bacilli may remain inactive for years and then activate when the client's resistance is lowered, as when a client is being treated for cancer. There's no such thing as tertiary infection, and superinfection doesn't apply in this case.

A client has active TB. Which of the following symptoms will he exhibit? A Chest and lower back pain B Chills, fever, night sweats, and hemoptysis C Fever of more than 104*F and nausea D Headache and photophobia

Question 12 Explanation: B Typical signs and symptoms are chills, fever, night sweats, and hemoptysis. Chest pain may be present from coughing, but isn't usual. Clients with TB typically have low-grade fevers, not higher than 102*F. Nausea, headache, and photophobia aren't usual TB symptoms.

Which of the following diagnostic tests is definitive for TB? A Chest x-ray B Mantoux test C Sputum culture D Tuberculin test

Question 13 Explanation: C The sputum culture for Mycobacterium tuberculosis is the only method of confirming the diagnosis. (Acid Fast Bacillis) Lesions in the lung may not be big enough to be seen on x-ray. Skin tests may be falsely positive or falsely negative and can only confirm exposure., not active TB.

A client with a positive Mantoux test result will be sent for a chest x-ray. For which of the following reasons is this done? A To confirm the diagnosis B To determine if a repeat skin test is needed C To determine the extent of the lesions D To determine if this is a primary or secondary infection

Question 14 Explanation: C If the lesions are large enough, the chest x-ray will show their presence in the lungs. Sputum culture confirms the diagnosis. There can be false-positive and false-negative skin test results. A chest x-ray can't determine if this is a primary or secondary infection.

A client with congestive heart failure has been receiving Digoxin (lanoxin). Which finding indicates that the medication is having a desired effect? A Increased urinary output B Stabilized weight C Improved appetite D Increased pedal edema

Question 15 Explanation: A Lanoxin slows and strengthens the contraction of the heart. An increase in urinary output shows that the medication is having a desired effect by eliminating excess fluid from the body. Answer B is incorrect because the weight would decrease. Answer C might occur but is not directly related to the question; therefore, it is incorrect. Answer D is incorrect because pedal edema would decrease, not increase.

A chest x-ray showed a client's lungs to be clear. His Mantoux test is positive, with a 10mm if induration. His previous test was negative. These test results are possible because: A He had TB in the past and no longer has it. B He was successfully treated for TB, but skin tests always stay positive C He's a "seroconverter", meaning the TB has gotten to his bloodstream D He's a "tuberculin converter," which means he has been infected with TB since his last skin test

Question 15 Explanation: D A tuberculin converter's skin test will be positive, meaning he has been exposed to an infected with TB and now has a cell-mediated immune response to the skin test. The client's blood and x-ray results may stay negative. It doesn't mean the infection has advanced to the active stage. Because his x-ray is negative, he should be monitored every 6 months to see if he develops changes in his x-ray or pulmonary examination. Being a seroconverter doesn't mean the TB has gotten into his bloodstream; it means it can be detected by a blood test.

A client with a positive skin test for TB isn't showing signs of active disease. To help prevent the development of active TB, the client should be treated with isoniazid, 300 mg daily, for how long? A 10 to 14 days B 2 to 4 weeks C 3 to 6 months D 9 to 12 months

Question 16 Explanation: D Because of the increased incidence of resistant strains of TB, the disease must be treated for up to 24 months in some cases, but treatment typically lasts for 9-12 months. Isoniazid is the most common medication used for the treatment of TB, but other antibiotics are added to the regimen to obtain the best results. ***THROW OUT. Positive test nothing done if disease isn't active, can lay dormant for years. (not certain, but I can't find in notes that you treat it)

A client with a productive cough, chills, and night sweats is suspected of having active TB. The physician should take which of the following actions? A Admit him to the hospital in respiratory isolation B Prescribe isoniazid and tell him to go home and rest C Give a tuberculin test and tell him to come back in 48 hours and have it read D Give a prescription for isoniazid, 300 mg daily for 2 weeks, and send him home

Question 17 Explanation: A The client is showing s/s of active TB and, because of the productive cough, is highly contagious. He should be admitted to the hospital, placed in respiratory isolation, and three sputum cultures should be obtained to confirm the diagnosis. He would most likely be given isoniazid and two or three other antitubercular antibiotics until the diagnosis is confirmed, then isolation and treatment would continue if the cultures were positive for TB. After 7 to 10 days, three more consecutive sputum cultures will be obtained. If they're negative, he would be considered non-contagious and may be sent home, although he'll continue to take the antitubercular drugs for 9 to 12 months.

A client is diagnosed with active TB and started on triple antibiotic therapy. What signs and symptoms would the client show if therapy is inadequate? A Decreased shortness of breath B Improved chest x-ray C Nonproductive cough D Positive acid-fast bacilli in a sputum sample after 2 months of treatment

Question 18 Explanation: D Continuing to have acid-fast bacilli in the sputum after 2 months indicated continued infection.

A 24-year-old client comes into the clinic complaining of right-sided chest pain and shortness of breath. He reports that it started suddenly. The assessment should include which of the following interventions? A Auscultation of breath sounds B Chest x-ray C Echocardiogram D Electrocardiogram (ECG

Question 21 Explanation: A Because the client is short of breath, listening to breath sounds is a good idea. He may need a chest x-ray and an ECG, but a physician must order these tests. Unless a cardiac source for the client's pain is identified, he won't need an echocardiogram.

Which of the following organisms most commonly causes community-acquired pneumonia in adults? A Haemiphilus influenzae B Klebsiella pneumoniae C Streptococcus pneumoniae D Staphylococcus aureus

Question 3 Explanation: C Pneumococcal or streptococcal pneumonia, caused by streptococcus pneumoniae, is the most common cause of community-acquired pneumonia. H. influenzae is the most common cause of infection in children. Klebsiella species is the most common gram-negative organism found in the hospital setting. Staphylococcus aureus is the most common cause of hospital-acquired pneumonia.

A police officer brings in a homeless client to the ER. A chest x-ray suggests he has TB. The physician orders an intradermal injection of 5 tuberculin units/0.1 ml of tuberculin purified derivative. Which needle is appropriate for this injection? A 5/8" to ½" 25G to 27G needle. B 1" to 3" 20G to 25G needle. C ½" to 3/8" 26 or 27G needle. D 1" 20G needle.

Question 34 Explanation: C Intradermal injections like those used in TB skin tests are administered in small volumes (usually 0.5 ml or less) into the outer skin layers to produce a local effect. A TB syringe with a ½" to 3/8" 26G or 27G needle should be inserted about 1/8" below the epidermis.

A 79-year-old client is admitted with pneumonia. Which nursing diagnosis should take priority? A Acute pain related to lung expansion secondary to lung infection B Risk for imbalanced fluid volume related to increased insensible fluid losses secondary to fever C Anxiety related to dyspnea and chest pain D Ineffective airway clearance related to retained secretions

Question 36 Explanation: D Pneumonia is an acute infection of the lung parenchyma. The inflammatory reaction may cause an outpouring of exudate into the alveolar spaces, leading to an ineffective airway clearance related to retained secretions.

A community health nurse is conducting an educational session with community members regarding TB. The nurse tells the group that one of the first symptoms associated with TB is: A A bloody, productive cough B A cough with the expectoration of mucoid sputum C Chest pain D Dyspnea

Question 37 Explanation: B One of the first pulmonary symptoms includes a slight cough with the expectoration of mucoid sputum.

A nurse evaluates the blood theophylline level of a client receiving aminophylline (theophylline) by intravenous infusion. The nurse would determine that a therapeutic blood level exists if which of the following were noted in the laboratory report? A 5 mcg/mL B 15 mcg/mL C 25 mcg/mL D 30 mcg/mL

Question 38 Explanation: B The therapeutic theophylline blood level range from 10-20 mcg/mL.

Isoniazid (INH) and rifampin (Rifadin) have been prescribed for a client with TB. A nurse reviews the medical record of the client. Which of the following, if noted in the client's history, would require physician notification? A Heart disease B Allergy to penicillin C Hepatitis B D Rheumatic fever

Question 39 Explanation: C Isoniazid and rifampin are contraindicated in clients with acute liver disease or a history of hepatic injury.

An elderly client with pneumonia may appear with which of the following symptoms first? A Altered mental status and dehydration B Fever and chills C Hemoptysis and dyspnea D Pleuritic chest pain and cough

Question 4 Explanation: A Fever, chills, hemoptysis, dyspnea, cough, and pleuritic chest pain are common symptoms of pneumonia, but elderly clients may first appear with only an altered mental status and dehydration due to a blunted immune response.

A client who is HIV+ has had a PPD skin test. The nurse notes a 7-mm area of induration at the site of the skin test. The nurse interprets the results as: A Positive B Negative C Inconclusive D The need for repeat testing

Question 41 Explanation: A The client with HIV+ status is considered to have positive results on PPD skin test with an area greater than 5-mm of induration. The client with HIV is immunosuppressed, making a smaller area of induration positive for this type of client.

A nurse is caring for a client diagnosed with TB. Which assessment, if made by the nurse, would not be consistent with the usual clinical presentation of TB and may indicate the development of a concurrent problem? A Nonproductive or productive cough B Anorexia and weight loss C Chills and night sweats D High-grade fever

Question 42 Explanation: D The client with TB usually experiences cough (non-productive or productive), fatigue, anorexia, weight loss, dyspnea, hemoptysis, chest discomfort or pain, chills and sweats (which may occur at night), and a low-grade fever.

A nurse is teaching a client with TB about dietary elements that should be increased in the diet. The nurse suggests that the client increase intake of: A Meats and citrus fruits B Grains and broccoli C Eggs and spinach D Potatoes and fish

Question 43 Explanation: A The nurse teaches the client with TB to increase intake of protein, iron, and vitamin C.

Which of the following would be priority assessment data to gather from a client who has been diagnosed with pneumonia? Select all that apply. A Auscultation of breath sounds B Auscultation of bowel sounds C Presence of chest pain D Presence of peripheral edema E Color of nail beds

Question 44 Explanation: A,C,E A respiratory assessment, which includes auscultating breath sounds and assessing the color of the nail beds, is a priority for clients with pneumonia. Assessing for the presence of chest pain is also an important respiratory assessment as chest pain can interfere with the client's ability to breathe deeply. Auscultating bowel sounds and assessing for peripheral edema may be appropriate assessments, but these are not priority assessments for the patient with pneumonia.

The cyanosis that accompanies bacterial pneumonia is primarily caused by which of the following? A Decreased cardiac output B Pleural effusion C Inadequate peripheral circulation D Decreased oxygenation of the blood

Question 46 Explanation: D A client with pneumonia has less lung surface available for the diffusion of gases because of the inflammatory pulmonary response that creates lung exudate and results in reduced oxygenation of the blood. The client becomes cyanotic because blood is not adequately oxygenated in the lungs before it enters the peripheral circulation.

A client with pneumonia has a temperature ranging between 101* and 102*F and periods of diaphoresis. Based on this information, which of the following nursing interventions would be a priority? A Maintain complete bedrest B Administer oxygen therapy C Provide frequent linen changes D Provide fluid intake of 3 L/day

Question 48 Explanation: D A fluid intake of at least 3 L/day should be provided to replace any fluid loss occurring as a result the fever and diaphoresis; this is a high-priority intervention.

Which of the following would be an appropriate expected outcome for an elderly client recovering from bacterial pneumonia? A A respiratory rate of 25 to 30 breaths per minute B The ability to perform ADL's without dyspnea C A maximum loss of 5 to 10 pounds of body weight D Chest pain that is minimized by splinting the ribcage

Question 49 Explanation: B An expected outcome for a client recovering from pneumonia would be the ability to perform ADL's without experiencing dyspnea. A respiratory rate of 25 to 30 breaths/minute indicates the client is experiencing tachypnea, which would not be expected on recovery. A weight loss of 5-10 pounds is undesirable; the expected outcome would be to maintain normal weight. A client who is recovering from pneumonia should experience decreased or no chest pain.

When auscultating the chest of a client with pneumonia, the nurse would expect to hear which of the following sounds over areas of consolidation? A Bronchial B Bronchovesicular C Tubular D Vesicular

Question 5 Explanation: A Chest auscultation reveals bronchial breath sounds over areas of consolidation. Bronchovesicular are normal over midlobe lung regions, tubular sounds are commonly heard over large airways, and vesicular breath sounds are commonly heard in the bases of the lung fields.

Which of the following symptoms is common in clients with TB? A Weight loss B Increased appetite C Dyspnea on exertion D Mental status changes

Question 50 Explanation: A TB typically produces anorexia and weight loss. Other signs and symptoms may include fatigue, low-grade fever, and night sweats.

The nurse obtains a sputum specimen from a client with suspected TB for laboratory study. Which of the following laboratory techniques is most commonly used to identify tubercle bacilli in sputum? A Acid-fast staining B Sensitivity testing C Agglutination testing D Dark-field illumination

Question 51 Explanation: A The most commonly used technique to identify tubercle bacilli is acid-fast staining. The bacilli have a waxy surface, which makes them difficult to stain in the lab. However, once they are stained, the stain is resistant to removal, even with acids. Therefore, tubercle bacilli are often called acid-fast bacilli.

Which of the following antituberculous drugs can cause damage to the eighth cranial nerve? A Streptomycin B Isoniazid C Para-aminosalicylic acid D Ethambutol hydrochloride

Question 52 Explanation: A Streptomycin is an aminoglycoside, and eight cranial nerve damage (ototoxicity) is a common side effect from ahminoglycosides **NOT ON TEST

The client experiencing eighth cranial nerve damage will most likely report which of the following symptoms? A Vertigo B Facial paralysis C Impaired vision D Difficulty swallowing

Question 53 Explanation: A The eighth cranial nerve is the vestibulocochlear nerve, which is responsible for hearing and equilibrium. Streptomycin can damage this nerve.

Which of the following family members exposed to TB would be at highest risk for contracting the disease? A 45-year-old mother B 17-year-old daughter C 8-year-old son D 76-year-old grandmother

Question 54 Explanation: D Elderly persons are believed to be at higher risk for contracting TB because of decreased immunocompetence. Other high-risk populations in the US include the urban poor, AIDS, and minority groups.

A client has a positive reaction to the PPD test. The nurse correctly interprets this reaction to mean that the client has: A Active TB B Had contact with Mycobacterium tuberculosis C Developed a resistance to tubercle bacilli D Developed passive immunity to TB

Question 56 Explanation: B A positive PPD test indicates that the client has been exposed to tubercle bacilli. Exposure does not necessarily mean that active disease exists.

INH treatment is associated with the development of peripheral neuropathies. Which of the following interventions would the nurse teach the client to help prevent this complication? A Adhere to a low cholesterol diet B Supplement the diet with pyridoxine (vitamin B6) C Get extra rest D Avoid excessive sun exposure

Question 57 Explanation: B INH competes with the available vitamin B6 in the body and leaves the client at risk for development of neuropathies related to vitamin deficiency. Supplemental vitamin B6 is routinely prescribed.

The nurse should include which of the following instructions when developing a teaching plan for clients receiving INH and rifampin for treatment for TB? A Take the medication with antacids B Double the dosage if a drug dose is forgotten C Increase intake of dairy products D Limit alcohol intake

Question 58 Explanation: D INH and rifampin are hepatotoxic drugs. Clients should be warned to limit intake of alcohol during drug therapy. Both drugs should be taken on an empty stomach. If antacids are needed for GI distress, they should be taken 1 hour before or 2 hours after these drugs are administered. Clients should not double the dosage of these drugs because of their potential toxicity. Clients taking INH should avoid foods that are rich in tyramine, such as cheese and dairy products, or they may develop hypertension.

The public health nurse is providing follow-up care to a client with TB who does not regularly take his medication. Which nursing action would be most appropriate for this client? A Ask the client's spouse to supervise the daily administration of the medications. B Visit the clinic weekly to ask him whether he is taking his medications regularly. C Notify the physician of the client's non-compliance and request a different prescription. D Remind the client that TB can be fatal if not taken properly.

Question 59 Explanation: A Directly observed therapy (DOT) can be implemented with clients who are not compliant with drug therapy. In DOT, a responsible person, who may be a family member or a health care provider, observes the client taking the medication. Visiting the client, changing the prescription, or threatening the client will not ensure compliance if the client will not or cannot follow the prescribed treatment.

A client with pneumonia develops dyspnea with a respiratory rate of 32 breaths/minute and difficulty expelling his secretions. The nurse auscultates his lung fields and hears bronchial sounds in the left lower lobe. The nurse determines that the client requires which of the following treatments first? A Antibiotics B Bed rest C Oxygen D Nutritional intake

Question 7 Explanation: C The client is having difficulty breathing and is probably becoming hypoxic. As an emergency measure, the nurse can provide oxygen without waiting for a physicians order. Antibiotics may be warranted, but this isn't a nursing decision. The client should be maintained on bedrest if he is dyspneic to minimize his oxygen demands, but providing additional will deal more immediately with his problem. The client will need nutritional support, but while dyspneic, he may be unable to spare the energy needed to eat and at the same time maintain adequate oxygenation.

A client has been treated with antibiotic therapy for right lower-lobe pneumonia for 10 days and will be discharged today. Which of the following physical findings would lead the nurse to believe it is appropriate to discharge this client? A Continued dyspnea B Fever of 102*F C Respiratory rate of 32 breaths/minute D Vesicular breath sounds in right base

Question 8 Explanation: D If the client still has pneumonia, the breath sounds in the right base will be bronchial, not the normal vesicular breath sounds. If the client still has dyspnea, fever, and increased respiratory rate, he should be examined by the physician before discharge because he may have another source of infection or still have pneumonia.

How is TB diagnosed? A. Chest X-ray B. Sample of sputum C. Skin test D. All of the above

The correct answer is D. All of the above. The first step in diagnosing TB is a skin test. This will show whether you have been exposed to M. tuberculosis. For the skin test, or PPD, a substance called tuberculin is injected just under the skin on your forearm. After 72 hours, you return to your health care provider, who checks the site for a positive or negative reaction. A positive skin test does not mean you have active TB, however. It simply means that you were exposed to the bacterium at some point or may have received the TB vaccine, called BCG. This vaccine is often routinely given in other countries, although it is not used in the U.S. A blood test is available in some health departments to check for TB exposure. Besides the skin or blood test, your doctor will look at your medical history and X-ray of the lungs to check for signs of active or old/healed tuberculosis. The doctor may also take samples of morning sputum and stain it to check for evidence of the TB bacterium. The sputum may also be sent for culture of the M. tuberculosis bacteria, although this may take several weeks to show a positive result.

What makes TB hard to diagnose? A. Symptoms aren't always obvious B. Symptoms come and go C. The disease may take years to become active D. A and B

The correct answer is C. The disease may take years to become active. Many infectious diseases cause illness very quickly. With TB, a person can be infected with the bacterium, but not become sick with TB disease. This is called latent TB infection. A person with latent TB is not contagious and cannot pass TB on to others. The other type of TB is active TB disease. Active TB may take several decades to occur - or it may develop soon after infection. Someone with active TB is contagious and can spread the disease to others. The active form may progress more quickly in people who have a weak immune system. Other infectious diseases have a latent period. These diseases can be hard to diagnose.

What are the symptoms of active TB? A. Weight loss B. Night sweats C. Loss of appetite D. All of the above

The correct answer is D. All of the above. Symptoms of active TB can depend on what part of the body has been infected. If active TB occurs in the lungs, the symptoms can include a cough lasting 3 weeks or more, pain in the chest area, a cough that brings blood or mucus, chills, fever, and fatigue. TB can also affect a person's brain, bones, kidneys, or spine.

Who is at risk for developing tuberculosis in this country? A. Alcoholics B. Migrant farm workers C. People with HIV D. All of the above

The correct answer is D. All of the above. TB is especially common in people with HIV. Latent or inactive TB is more likely to become an active TB infection in someone with HIV. Other groups at risk for TB include the urban poor, intravenous drug users, homeless people, prison inmates, people born overseas, and health care workers.

What is causing TB to become a serious public health problem? A. The rise in number of people with HIV infection B. An increasing number of immigrants from areas of the world where TB is prevalent C. An increasing number of homeless people D. More people failing to finish their TB treatment E. All of the above

The correct answer is E. All of the above. The number of TB cases began dropping in the 1940s and 1950s, when antibiotics were first used to treat the disease. But after 1985, the number of TB cases started to rise again. One factor was the AIDS epidemic. People with HIV infection are at high risk for developing active TB. People who must live in crowded homeless shelters are at higher risk for developing active TB. People who don't finish their TB treatment are at risk for developing drug-resistant TB.

Who should receive a vaccine for TB? A. Infants B. Teens C. Adults under age 65 D. Adults over age 65 E. None of the above

The correct answer is E. None of the above. Although a vaccine is available, it is not recommended in this country because the rate of TB infection is low and because vaccinating people would make the TB skin test less useful. Someone vaccinated against TB may show a positive skin test even if not infected, especially in the few years following vaccination. Infants who live in areas of the world where TB is prevalent should be vaccinated against TB. The vaccine does not work very well in adults.

A client with pneumonia has a temperature of 102.6*F (39.2*C), is diaphoretic, and has a productive cough. The nurse should include which of the following measures in the plan of care? A Position changes q4h B Nasotracheal suctioning to clear secretions C Frequent linen changes D Frequent offering of a bedpan

uestion 45 Explanation: C Frequent linen changes are appropriate for this client because of diaphoresis. Diaphoresis produces general discomfort. The client should be kept dry to promote comfort. Position changes need to be done every 2 hours. Nasotracheal suctioning is not indicated with the client's productive cough. Frequent offering of a bedpan is not indicated by the data provided in this scenario.


Kaugnay na mga set ng pag-aaral

Mastering Chemistry Chapter 1, Sec 1 & 2

View Set

10. Expansion of Civil Rights / Supreme Court Cases

View Set

Chapter 1 Review-Financial Accounting

View Set

Chapter 6CONNECT assignment (27)

View Set

Which of the following structures is included in the respiratory portion of the respiratory tract?

View Set